Algèbres bilinéaire et hermitienne

Exercices du dossier Algèbres bilinéaire et hermitienne

Accordéon
Titre
Solution
Texte

Dimension, bases et applications linéaires
Par Patrice Lassère le 7 novembre 2022 22:14

Compétons une base \(\{e_1,e_2,e_3\}\) de \(E_1\) pour obtenir une base \(\{e_1,e_2,e_3,e_4,e_5,e_6\}\) de \(E_2\) ; enfin, complétons la encore une fois pour obtenir une base \(\mathscr B=\{e_1,\dots,e_{10}\}\) de \(\mathbb R^{10}\). Avec ce choix, la matrice d’un endomorphisme \(T\in\mathscr E\) sera de la forme \[{\rm{mat}}(T,\mathscr B)=\begin{pmatrix} ?&?&?&?&?&?&?&?&?&?\\ ?&?&?&?&?&?&?&?&?&?\\ ?&?&?&?&?&?&?&?&?&?\\ 0&0&0&?&?&?&?&?&?&?\\ 0&0&0&?&?&?&?&?&?&?\\ 0&0&0&?&?&?&?&?&?&?\\ 0&0&0&0&0&0&?&?&?&?\\ 0&0&0&0&0&0&?&?&?&?\\ 0&0&0&0&0&0&?&?&?&?\\ 0&0&0&0&0&0&?&?&?&?\\ \end{pmatrix}\] et par conséquent \(\dim_\mathbb R\mathscr E= 9+18+40=67\).


Matrices et réduction *

7 novembre 2022 22:14 — Par Patrice Lassère

Soient \(A=\begin{pmatrix} 1&a\\0&1\end{pmatrix},\ B=\begin{pmatrix} b&c\\0&b\end{pmatrix},\ C\in M_2(\mathbb R)\). À quelle condition la matrice \(\begin{pmatrix} A&C\\0&B\end{pmatrix}\) est-elle diagonalisable ?



[ID: 2293] [Date de publication: 7 novembre 2022 22:14] [Catégorie(s): Algèbres bilinéaire et hermitienne ] [ Nombre commentaires: 0] [nombre d'éditeurs: 1 ] [Editeur(s): Emmanuel Vieillard-Baron ] [nombre d'auteurs: 1 ] [Auteur(s): Patrice Lassère ]
Accordéon
Titre
Solution
Texte

Matrices et réduction
Par Patrice Lassère le 7 novembre 2022 22:14

Le résultat suivant est essentiel : soit \(M\in M_n(\mathbb K)\) une matrice donc le polynôme caractéristique est scindé et s’écrit \(\chi_M=(X-\lambda_1)^{\alpha_1}\dots (X-\lambda_d)^{\alpha_d}\) où les \(\lambda_i\) sont les valeurs propres deux à deux distinctes de \(M\) de multiplicités \(\alpha_i\in\mathbb N^\star\). Alors \(M\) est diagonalisable si, et seulement si, \(M\) annule \((X-\lambda_1)\dots (X-\lambda_d)\).

Ici \(\chi_M=(X-1)^2(X-b)^2\). Distinguons deux cas :

-Si \(b=1\). \(M\) est diagonalisable si, et seulement si, \(M=I_4\) soit \(a=c=0\) et \(C=0\).

-Si \(b\neq 1\). Dans ce cas, \(M\) est diagonalisable si, et seulement si, \((M-I_4)(M-bI_4)=0\). Un calcul direct montre que cette dernière condition équivaut à \(a=c=0\). En résumé, \(M\) est diagonalisable si, et seulement si \[\left(\,a=c=0\,\right)\quad\text{et}\quad \left(\,b\neq 1\ \text{ ou }\ C=0\right).\]


Une équation matricielle dans \(M_2(\mathbb C)\) *

7 novembre 2022 22:14 — Par Patrice Lassère

Montrer que l’équation \[X^r=\begin{pmatrix} 0&1\\0&0\end{pmatrix}\] n’a pas de solutions dans \(M_2(\mathbb C)\) pour tout entier \(r\geq 2\).



[ID: 2295] [Date de publication: 7 novembre 2022 22:14] [Catégorie(s): Algèbres bilinéaire et hermitienne ] [ Nombre commentaires: 0] [nombre d'éditeurs: 1 ] [Editeur(s): Emmanuel Vieillard-Baron ] [nombre d'auteurs: 1 ] [Auteur(s): Patrice Lassère ]
Accordéon
Titre
Solution
Texte

Une équation matricielle dans \(M_2(\mathbb C)\)
Par Patrice Lassère Emmanuel Vieillard-Baron le 7 novembre 2022 22:14

Supposons qu’il existe \(r\geq 2\) et \(A\in M_2(\mathbb C)\) tels que \(A^r=\begin{pmatrix} 0&1\\0&0\end{pmatrix}\). Alors \(A^{2r}=\begin{pmatrix} 0&0\\0&0\end{pmatrix}\), et le polynôme caractéristique \(\chi_A(x)=ax^2+bx+c\) de \(A\) divise donc \(x^{2r}\) ; ceci implique \(c=b=0\), soit \(\chi_A(x)=x^2\) et (Cayley-Hamilton) \(A^2=\begin{pmatrix} 0&0\\0&0\end{pmatrix}\). Ainsi (comme \(r\geq 2\)) \[\begin{pmatrix} 0&1\\0&0\end{pmatrix}=A^r=A^2A^{r-2}=\begin{pmatrix} 0&0\\0&0\end{pmatrix}\] ce qui est absurde : cette équation est bien sans solutions.


Histoire de matrices nilpotentes *

7 novembre 2022 22:15 — Par Patrice Lassère

[rms]-(2003/04).

Soit \(A\) et \(B\) dans \({M}_n(\mathbb{K})\) telles que \(ABAB=0\). A-t-on \(BABA=0\) ?



[ID: 2297] [Date de publication: 7 novembre 2022 22:15] [Catégorie(s): Algèbres bilinéaire et hermitienne ] [ Nombre commentaires: 0] [nombre d'éditeurs: 1 ] [Editeur(s): Emmanuel Vieillard-Baron ] [nombre d'auteurs: 1 ] [Auteur(s): Patrice Lassère ]
Accordéon
Titre
Solution
Texte

Histoire de matrices nilpotentes
Par Patrice Lassère Emmanuel Vieillard-Baron le 7 novembre 2022 22:15

La réponse est oui si \(n\leqslant 2\), non si \(n>2\).

-Si \(n=1\): c’est clair. Si \(n=2\), on a \((BA)^3=B(AB)^2A=0\) donc \(BA\) est nilpotente; comme elle est de taille \(2\), son indice est inférieur à \(2\) i.e. \((BA)^2=0\).

-Pour \(n=3\), \(BA\) est nilpotente d’indice au plus \(3\) et \(AB\) d’indice de nilpotence au plus \(2\). On va chercher \(A,B\) pour que \(AB\) soit d’indice \(2\) et \(BA\) soit d’indice \(3\) et plus précisément telles que, par exemple, \[BA=N=\begin{pmatrix} 0&1&0\\ 0&0&1\\ 0&0&0\end{pmatrix}.\] Comme \(\mathop{\rm Ker}(A)\) doit être une droite (\(A\) n’est pas inversible et de rang supérieur à celui de \(N\), donc de rang \(2\)) incluse dans le noyau de \(N=BA\), \(\mathop{\rm Ker}(A)\) est engendré par \((1,0,0)\). De même \(\mathop{\rm Im}(B)\) doit être un plan qui doit contenir l’image de \(N\) i.e. le plan engendré par \((1,0,0)\) et \((0,1,0)\).

On est donc amené à chercher sous les formes \(A=\begin{pmatrix} 0&a&a'\\ 0&b&b'\\ 0&c&c'\end{pmatrix}\) et \(B=\begin{pmatrix} d&e&f\\ d'&e'&f'\\ 0&0&0\end{pmatrix}\). Un peu de calcul montre que \(A=\begin{pmatrix} 0&1&0\cr 0&0&1\cr 0&0&0\end{pmatrix}\) et \(B=\begin{pmatrix} 1&0&0\\ 0&1&0\\ 0&0&0\end{pmatrix}\) conviennent.

-Pour \(n>3\) il suffit de border les matrices précédentes par des zéros.


Autour du commutant *

7 novembre 2022 22:15 — Par Patrice Lassère

[rms]-(1998).

Déterminer la structure de l’ensemble \(\mathscr E\) des endomorphismes \(\varphi\in \mathscr L\left( M_n(\mathbb R)\right)\) vérifiant \[\varphi({ }^t\! B)={ }^t\!\varphi(B),\quad\forall\,B\in M_n(\mathbb R).\]



[ID: 2299] [Date de publication: 7 novembre 2022 22:15] [Catégorie(s): Algèbres bilinéaire et hermitienne ] [ Nombre commentaires: 0] [nombre d'éditeurs: 1 ] [Editeur(s): Emmanuel Vieillard-Baron ] [nombre d'auteurs: 1 ] [Auteur(s): Patrice Lassère ]
Accordéon
Titre
Solution
Texte

Autour du commutant
Par Patrice Lassère le 7 novembre 2022 22:15

Si on désigne par \(T\) l’endomorphisme \(M\mapsto\,{ }^t\!M\) de \(M_n(\mathbb R)\), \(\mathscr E\) n’est rien d’autre que le commutant de \(T\) et c’est donc une sous-algèbre de \(\mathscr L\left( M_n(\mathbb R)\right)\). Soit \(\mathscr S_n\) (resp. \(\mathscr A_n\)) l’espace vectoriel des matrices symétriques (resp. antisymétrique). Puisque \(M_n(\mathbb R)=\mathscr S_n\oplus\mathscr A_n\), un endomorphisme \(\varphi\) vérifiant \(\varphi\circ T=T\circ\varphi\) doit laisser stable les deux sous-espaces propres de \(T\) que sont \(\mathscr S_n\) et \(\mathscr A_n\). Mais inversement, un endomorphisme laissant stable \(\mathscr S_n\) et \(\mathscr A_n\) satisfait (avec les notations évidentes) à \[{ }^t\!\varphi(B)={ }^t\!\varphi(S+A)={ }^t\!\varphi(S)+{ }^t\!\varphi(A)=\varphi(S)-\varphi(A)=\varphi(S-A)=\varphi\left( { }^t\!(S+A)\right) =\varphi({ }^t\!B)\] ce qui montre que \(\varphi\in\mathscr E\). L’algèbre \(\mathscr E\) est donc constituée des endomorphisme de \(M_n(\mathbb R)\) qui laissent stable les sous-espaces \(\mathscr S_n\) et \(\mathscr A_n\). Il existe donc un isomorphisme évident \[\mathscr E\longrightarrow \mathscr L(\mathscr S_n)\times\mathscr L(\mathscr A_n)\] qui à \(\varphi\) associe le couple \(\left(\varphi_{/\mathscr S_n},\varphi_{/\mathscr A_n} \right)\). Il en résulte que \[\text{dim}(\mathscr E)=\left( \dfrac{n(n+1)}{2}\right)^2+\left( \dfrac{n(n-1)}{2}\right)^2=\dfrac{n^2(n^2+1)}{2}.\]

Remarque : on retrouve dans la dernière formule le résultat connu mais plus délicate à établir, à savoir que la dimension du commutant est toujours une somme de carrés \(p_1^2+\dots +p_k^2\in\left\lbrace d,d+1,\dots, d^2\right)\) (ici \(d=n^2\)) telle que \(p_1+\dots+p_k=d\).


Étude \(A\mapsto A^3\) dans \(M_3(\mathbb R)\) *

7 novembre 2022 22:15 — Par Patrice Lassère

Déterminer l’image de l’application \(\varphi\) de \(M_3(\mathbb R)\) dans \(M_3(\mathbb R)\) définie par \(\varphi(A)=A^3\).



[ID: 2301] [Date de publication: 7 novembre 2022 22:15] [Catégorie(s): Algèbres bilinéaire et hermitienne ] [ Nombre commentaires: 0] [nombre d'éditeurs: 1 ] [Editeur(s): Emmanuel Vieillard-Baron ] [nombre d'auteurs: 1 ] [Auteur(s): Patrice Lassère ]
Accordéon
Titre
Solution
Texte

Étude \(A\mapsto A^3\) dans \(M_3(\mathbb R)\)
Par Patrice Lassère le 7 novembre 2022 22:15

Nous allons vérifier que cette image est constituée de toutes les matrices \(A\in M_3(\mathbb R)\) sauf celles admettant \(0\) comme valeur propre multiple et qui ne sont pas diagonalisables.

Soit \(B\in M_3(\mathbb R)\) et cherchons \(A\in M_3(\mathbb R)\) vérifiant \(\varphi(A)=A^3\). On distingue plusieurs cas pour \(B\)

-\(B\) est diagonalisable sur \(\mathbb R\).

Il existe alors une base \(\left\lbrace u,v,w\right\rbrace\) de \(\mathbb R^3\), des réels \(\alpha,\beta,\gamma\) tels que \[Bu=\alpha u,\quad Bv=\beta v,\quad Bw=\gamma w.\] \(\lambda,\mu,\nu\in\mathbb R\) désignant les racines cubiques des valeurs propres \(\alpha,\beta,\gamma\), définissons la matrice \(A\in M_3(\mathbb R)\) par \[Au=\lambda u,\quad Av=\mu v,\quad Aw=\nu w.\] \(A\) est bien réelle et vérifie \(A^3=B\) et \(B\) admet bien un antécédent par \(\varphi\).

-\(B\) possède une valeur propre non réelle.

Les valeurs propres de \(B\) sont donc un réel \(\alpha\) et deux complexes non réels conjugués \(\omega\) et \(\overline{\omega}\). Il existe un vecteur réel non nul \(u\) et un vecteur complexe \(z\) non nul tels que \[Bu=\alpha u,\quad Bz=\omega z,\quad\text{et par suite},\quad B\overline{z}=\overline{\omega}\overline{z}.\] Notons \(\lambda\) le réel racine cubique de \(\alpha\) et soit \(\theta\) une racine cubique de \(\omega\). La matrice \(A\) définie dans la base \((u,z,\overline{z})\) de \(\mathbb C^3\) par \[Au=\lambda u,\quad Az=\theta z,\quad A\overline{z}=\overline{\theta}\overline{z}\] vérifie \(A^3=B\). En outre \(\overline{A}\) envoie \(u\) sur \(\lambda u\), \(z\) sur \(\overline{\theta}z\) et \(\overline{z}\) sur \(\theta\overline{z}\). \(A\) est donc réelle et et \(B\) admet bien un antécédent par \(\varphi\).

-\(B\) possède une valeur propre réelle non nulle \(\lambda\) d’ordre \(2\) et n’est pas diagonalisable.

Les valeurs propres de \(B\) sont \(\lambda,\lambda,\mu\)\(\mu\) est un autre réel. Posons \(\alpha=\lambda^{1/3},\ \beta=\mu^{1/3}\), on a donc \[\mathbb R^3=\ker(B-\lambda I_3)^2\oplus\ker(B-\mu I_3)\] La dimension de \(\ker(B-\lambda I_3)\) n’est pas deux sinon \(B\) serait diagonalisable, elle vaut donc \(1\). Puisque \(\dim\ker(B-\lambda I_3)^2=2,\ \dim\ker(B-\lambda I_3)=1\), considérons \(u\in\ker(B-\lambda I_3)^2\setminus\ker(B-\lambda I_3),\ v=(B-\lambda I_3)u\) et \(w\) un vecteur non nul de \(\ker(B-\mu I_3)\). on a ainsi construit une base \((u,v,w)\) de \(\mathbb R^3\) qui vérifie \[Bu=\lambda u+v,\quad Bv=\lambda v,\quad Bw=\mu w.\] Une matrice \(A\in M_3(\mathbb R^3)\) vérifiant pour un certain réel \(c\) \[Au=\alpha u+cv,\quad Av=\alpha v,\quad Aw=\beta w\] vérifiera \[A^3u=\lambda u+3\alpha^2cv,\quad A^3v=\lambda v,\quad A^3w=\mu w\] de sorte que si \(c=\dfrac{1}{3\alpha^2}\) : \(A^3=B\).

-\(B\) possède une valeur propre non nulle \(\lambda\) d’ordre \(3\) et n’est pas diagonalisable

Dans ce cas \(B=\lambda(I_3+N)\)\(N\) est une matrice nilpotente non nulle. Posons \(\alpha=\lambda^{1/3}\) et cherchons \(A\) sous la forme \(A=\alpha(I_3+M)\) avec \(M\) nilpotente. On a \(M^3=0\) donc \(A^3=\lambda (I_3+3M+3M^2)\) et tout se ramène à l’équation \(3M+3M^2=N\) qui est vérifiée par \(M=\dfrac{1}{9}(3N-N^2)\). Une fois de plus \(B\) admet un antécédent.

-\(B\) admet \(0\) comme valeur propre d’ordre \(2\) ou \(3\) et n’est pas diagonalisable.

Si l’équation \(A^3=B\) admet une solution, \(A\) admet aussi \(à\) comme vaelur propre d’ordre \(2\) ou \(3\). Si \(0\) est valeur propre d’ordre \(3\) alors \(A\) est nilpotente, \(A^3=0\), ce qui est absurde puisque \(B\) n’est pas diagonalisable. Supposons donc que \(O\) soit valeur propre d’ordre \(2\) de \(A\) et notons \(\alpha\) l’autre valeur propre (réelle). \(\mathbb R^3=\ker(A^2)\oplus\ker(A-\alpha I_3)\) mais \(A^3=B\) implique \(Bx=0,\ \forall\,x\in\ker(A^2)\) soit \(\mathbb R^3=\ker(B)\oplus\ker(B-\alpha^3I_3)\) : \(B\) est alors diagonalisable ce qui est exclut : tous les cas sont épuisés et la conclusion annoncée s’impose.


Réduction des endomorphismes *

7 novembre 2022 22:15 — Par Patrice Lassère

Pour \(n\in\mathbb N^\star,\ A\in M_n(\mathbb C)\) on définit la matrice \(B\in\mathbb M_{2n}(\mathbb C)\) par

\[B=\begin{pmatrix} A&A\\ 0&A\end{pmatrix}\]

Montrer que \(B\) est diagonalisable si, et seulement si \(A=0\).



[ID: 2303] [Date de publication: 7 novembre 2022 22:15] [Catégorie(s): Algèbres bilinéaire et hermitienne ] [ Nombre commentaires: 0] [nombre d'éditeurs: 1 ] [Editeur(s): Emmanuel Vieillard-Baron ] [nombre d'auteurs: 1 ] [Auteur(s): Patrice Lassère ]
Accordéon
Titre
Solution
Texte

Réduction des endomorphismes
Par Patrice Lassère le 7 novembre 2022 22:15

Si \(A=0\) alors \(B=0\) est diagonalisable. Réciproquement, vu la forme de \(A\) la polynôme caractéristique de \(B\) vérifie \[\chi_B=\left( \chi_A\right)^2\] et on a

\[\forall\,k\in\mathbb N,\quad B^k=\begin{pmatrix} A^k&kA^k\\ 0&A^k\end{pmatrix}\]

qui implique

\[P(B)=\begin{pmatrix} P(A)&AP'(A)\\ 0&P(A)\end{pmatrix},\quad\forall\,P\in\mathbb C[X].{(\diamondsuit)}\]

En particulier, le polynôme minimal de \(B\) vérifie

\[\mu_B(A)=0\]

et

\[\mu_A\quad\text{ divise }\quad\mu_B.{(\text{$\star$})}\]

Supposons \(B\) diagonalisable, \(\mu_B\) ne possède que des racines simples et comme \(A\) et \(B\) ont même ensemble valeurs propres avec (\(\star\)) nous avons \(\mu_A=\mu_B\) qui avec \((\diamondsuit)\) implique

\[A\mu'_A(A)=0.\]

\(\mu_A\) divise donc \(X\mu'_A\), les deux polynômes \(\mu_A\) et \(X\mu'_A\) étant de même degré \(d\)

\[d\mu_A=X\mu'_A\]

soit \(\mu_A=X^d\). Mais \(\mu_A=\mu_B\) n’a que des racines simples, donc \(d=1\), \(\mu_A(X)=X\) et finalement \(A=0\).


Accordéon
Titre
Solution
Texte

Encore deux démonstrations du Théorème de Cayley-Hamilton
Par Patrice Lassère le 7 novembre 2022 22:15
  1. Si \(A\in M_n(\mathbb C)\) est diagonalisable, il existe \(G\in GL_n(\mathbb C)\) telle que

    \[A=G \begin{pmatrix} \lambda_1&0&\dots&0\\ 0&\lambda_2&\dots&0\\ \vdots&\vdots&\ddots&\vdots\\ 0&\dots&0&\lambda_n \end{pmatrix} G^{-1}\]

    si bien que

    \[P_A(A)=G \begin{pmatrix} P_A(\lambda_1)&0&\dots&0\\ 0&P_A(\lambda_2)&\dots&0\\ \vdots&\vdots&\ddots&\vdots\\ 0&\dots&0&P_A(\lambda_n) \end{pmatrix} G^{-1}=0.\]

    L’application continue

    \[M_n(\mathbb C)\ni M\mapsto P_M(M)\in M_n(\mathbb C)\]

    est donc nulle sur \(\mathscr D_n(\mathbb C)\), partie dense de \(M_n(\mathbb C)\) (c.f. exercice 69) : elle est donc identiquement nulle.

  2. On cherche à montrer que \(P_A(A)x=0\) pour tout vecteur \(x\). Soit donc \(x\) un vecteur non nul de \(\Bbb{C}^n\) (si \(x=0\) il n’y a rien à démontrer), notons \(\mathscr{E}_x\) le plus petit (au sens de l’inclusion) sous-espace stable par \(A\) de \(\Bbb{C}^n\) contenant \(x\). \(\mathscr{E}_x\) est de dimension \(1\leq d\leq n\), et admet pour base \(\{ x,Ax,\dots ,A^{d-1}x \}.\)

    Il existe donc \(a_{n-1},\dots ,a_1,a_0\) dans \(\Bbb C\) tels que

    \[A^{d}x \ =\ a_{d-1}A^{d-1}x+\dots +a_1Ax+a_0x.{(\bigstar )}\]

    Complétons alors la famille libre \(\{x,Ax,\dots ,A^{d-1}x\}\) pour obtenir une base de \(\Bbb C ^n\) de la forme

    \[\{ x,Ax,\dots ,A^{d-1}x,e_{d+1},\dots ,e_n \}\]

    Dans cette base la matrice \(A\) est de la forme \[\begin{pmatrix} C_P&?\\ 0& B \end{pmatrix}\]\(B \in M_{n-d}(\Bbb {C})\) et \(C_P\) est la matrice compagnon du polynôme \(P(X) \,=\,X^{d}-a_{d-1}X^{d-1}-\dots -a_1X-a_0\ \ \) i.e. :

    \[C_P \ = \begin{pmatrix} 0 & 0 & \dots & 0 & a_0 \\ 1 & 0 & \dots & 0 & a_1 \\ 0 & 1 & \dots & 0 & a_2 \\ \vdots & \ddots & \ddots & \vdots & \vdots \\ 0 & \dots & 0 & 1 & a_{n-1} \end{pmatrix}.\] on a donc \(P_A(X) = P_{C_P}(X)P_B(X)\), mais alors

    \[\begin{aligned} P_A(A)x &= P_{C_P}(A)P_B(A)x \\ &= P_B(A)P_{C_P}(A)x \\ &= P_B(A)(A^{d}x - a_{d-1}A^{d-1}x-\dots -a_1Ax-a_0x )= 0 \ \text{ vu}\quad (\bigstar ) \end{aligned}\]

    dans la seconde inégalité les deux endomorphismes \(P_{C_P}(A)\) et \(P_B(A)\) commutent légitimement comme polynômes en \(A\). Ainsi \(P_A(A)x=0,\ \forall x\in \Bbb C^n\) i.e. \(P_A\equiv 0\).

    - on notera l’extrême simplicité de cette dernière preuve (niveau première année) qui est valable dans un contexte bien plus général que la précédente.

    - Pour une autre démonstration moins classique voir l’exercice 94 et pour un joli bestiaire de preuves de Cayley-Hamilton consulter /httpblabla?????????


Étude de \(M_3(\mathbb R)\ni B\mapsto AB\) *

7 novembre 2022 22:15 — Par Patrice Lassère

Pour \(A\in M_3(\mathbb R)\), considérons l’endomorphisme

\[\varphi_A\ :\ B\in M_3(\mathbb R)\mapsto \varphi_A(B)=AB\in M_3(\mathbb R).\]

Si le déterminant de \(A\) est \(32\) et son polynôme minimal \((t-4)(t-2)\), quelle est la trace de \(\varphi_A\) ?



[ID: 2307] [Date de publication: 7 novembre 2022 22:15] [Catégorie(s): Algèbres bilinéaire et hermitienne ] [ Nombre commentaires: 0] [nombre d'éditeurs: 1 ] [Editeur(s): Emmanuel Vieillard-Baron ] [nombre d'auteurs: 1 ] [Auteur(s): Patrice Lassère ]
Accordéon
Titre
Solution
Texte

Étude de \(M_3(\mathbb R)\ni B\mapsto AB\)
Par Patrice Lassère le 7 novembre 2022 22:15

\(\pi_A(t)=(t-4)(t-2)\) est scindé à racines simples : \(A\) est diagonalisable et admet comme valeurs propres \(4\) et \(2\) de multiplicités respectives \(\alpha\) et \(\beta\) ; le déterminant est donc \(\text{det}(A)=32=2^\alpha4^\beta\) soit \(\alpha=1\) et \(\beta=2\) (le polynôme caractéristique de \(A\) est \(P_A(t)=-(t-4)^2(t-2)\)...). Soit \(\{v_1,v_2,v_3\}\) une base de vecteurs propres de \(A\) avec \(\ker(A-4I_3)=\text{vect}\{v_1,v_2\},\ \ker(A-2I_3)=\text{vect}\{v_3\}\) et considérons les neuf matrices \(E_{i,j}\in M_3(\mathbb R)\ 1\leq i,j\leq 3\) définies comme suit : la ième colonne de la matrice \(E_{i,j}\) est \(v_j\) les deux autres colonnes sont constituées de zéros. Les vecteurs \(v_1,v_2,v_3\) formant une base de \(\mathbb R^3\), les neuf matrices \(E_{i,j}\) sont linéairement indépendantes dans \(M_3(\mathbb R)\) : c’est donc une base de \(M_3(\mathbb R)\). Par ailleurs ce sont des vecteurs propres de \(\varphi_A\) car un calcul élémentaire nous montre que \(\varphi_A(E_{i,j})=A E_{i,j}=\lambda_j E_{i,j}\) avec \(\lambda_1=\lambda_2=4\) et \(\lambda_3=2\). Ainsi \(M_3(\mathbb R)\) possède une base de vecteurs propres : \(\varphi_A\) est donc diagonalisable et sa trace est \(\text{trace}(A)=6 .4+3. 2=30.\)


\(A^5+A^3+A=3I_d\) dans \(M_d(\mathbb C)\) *

7 novembre 2022 22:15 — Par Patrice Lassère

Soit \(A\in M_d(\mathbb C)\) une matrice à valeurs propres réelles vérifiant \(A^5+A^3+A=3I_d\), Montrer que \(A=I_d\).



[ID: 2309] [Date de publication: 7 novembre 2022 22:15] [Catégorie(s): Algèbres bilinéaire et hermitienne ] [ Nombre commentaires: 0] [nombre d'éditeurs: 1 ] [Editeur(s): Emmanuel Vieillard-Baron ] [nombre d'auteurs: 1 ] [Auteur(s): Patrice Lassère ]
Accordéon
Titre
Solution
Texte

\(A^5+A^3+A=3I_d\) dans \(M_d(\mathbb C)\)
Par Patrice Lassère le 7 novembre 2022 22:15

Le polynôme \(p(t)=t^5+t^3+t-3\) est annulé par \(A\) donc \(\pi_A\) divise \(p\). Les valeurs propres de \(A\) sont réelles et son polynôme minimal ne possède que des racines réelles, mais \(p'(t)=5t^4+3t^2+1>0\) sur \(\mathbb R\) donc \(p\) ne possède qu’une racine réelle (il est de degré impair donc en possède au moins une mais il ne peut en avoir deux car sinon, par Rolle, \(p'\) en aurait une...) et il n’est pas difficile de voir que \(p(1)=0\) cette racine est donc \(1\), en outre \(p'(1)\ne 0\) ce n’est donc pas une racine multiple et par conséquent \(p(t)=(t-1)q(t)\)\(q\) est sans racines réelles. \(\pi_A\) divise donc le polynôme irréductible \(t-1\) i.e. \(\pi_A(t)=t-1\) et \(A+I_d\).


Polynôme minimal et dimension du noyau *

7 novembre 2022 22:15 — Par Patrice Lassère

Si le polynôme minimal d’un endormorphisme \(\varphi\) sur un espace vectoriel de dimension \(7\) est \(\pi_\varphi(t)=t^2\), quelles sont les valeurs possibles de \(\text{dim}\left(\ker(\varphi)\right)\) ?



[ID: 2311] [Date de publication: 7 novembre 2022 22:15] [Catégorie(s): Algèbres bilinéaire et hermitienne ] [ Nombre commentaires: 0] [nombre d'éditeurs: 1 ] [Editeur(s): Emmanuel Vieillard-Baron ] [nombre d'auteurs: 1 ] [Auteur(s): Patrice Lassère ]
Accordéon
Titre
Solution
Texte

Polynôme minimal et dimension du noyau
Par Patrice Lassère le 7 novembre 2022 22:15

\(\pi_\varphi(t)=t^2\), \(0\) est donc valeur propre de \(\varphi\) : \(\text{dim}\left(\ker(\varphi)\right)\geq 1\), de l’autre côté puisque \(\pi_\varphi(t)\ne t\), \(\varphi\) n’est pas l’endomorphisme nul i.e. \(\text{dim}\left(\ker(\varphi)\right)\leq 6\) et enfin \(\text{dim}\left(\ker(\varphi^2)\right)\leq 7\).

Soit \(\psi\), la restriction de \(\varphi\) à \(\ker(\varphi^2)\), par le théorème du rang :

\[\begin{aligned} \text{dim}\left(\ker(\varphi^2)\right)&=\text{dim}\left(\ker(\psi)\right)+\text{ dim}\left(\text{im}(\psi)\right) \\ &\leq \text{dim}\left(\ker(\varphi)\right)+ \text{dim}\left(\ker(\varphi )\right)=2\,\text{dim}\left(\ker(\varphi)\right) \end{aligned}\] \(\rightsquigarrow\) l’inégalité résultant des inclusions faciles à vérifier : \[\ker(\psi)\subset\ker(\varphi)\quad\&\quad \text{im}(\psi)\subset\ker(\varphi).\] Ainsi, nous avons \[7=\text{dim}\left(\ker(\varphi^2)\right)\leq 2\,\text{dim}\left(\ker(\varphi) \right)\ \&\ \text{dim}\left(\ker(\varphi)\right)\in\{1,\dots,6\}\] soit \[\text{dim}\left(\ker(\varphi)\right) \in\{4,5,6\}\] et les trois exemples ci-dessous montrent que les valeurs \(4,5,6\) sont toujours possibles (\(\{e_1,\dots,e_7\}\) désigne une quelconque base de notre espace) :

\(\rightsquigarrow\)\(\text{dim}\left(\ker(\varphi)\right)=6\), définir \(\varphi\) par \(\varphi(e_7)=e_6\) et \(\varphi(e_j)=0\) pour \(j<7\).

\(\rightsquigarrow\)\(\text{dim}\left(\ker(\varphi)\right)=5\), définir \(\varphi\) par \(\varphi(e_7)=e_5,\varphi(e_6)=e_4\) et \(\varphi(e_j)=0\) pour \(j<6\).

\(\rightsquigarrow\)\(\text{dim}\left(\ker(\varphi)\right)=4\), définir \(\varphi\) par \(\varphi(e_7)=e_4, \varphi(e_6)=e_3, \varphi(e_5)=e_2\) et \(\varphi(e_j)=0\) pour \(j<5\).

 


Étude de \(\varphi\,:\,A\in M_n(\mathbb R)\longmapsto \varphi(A)=-A+\text{tr}(A)I_n.\) *

7 novembre 2022 22:15 — Par Patrice Lassère

Étudier la diagonalisabilité de

\[\varphi\,:\,A\in M_n(\mathbb R)\longmapsto \varphi(A)=-A+\text{tr}(A)I_n.\]



[ID: 2313] [Date de publication: 7 novembre 2022 22:15] [Catégorie(s): Algèbres bilinéaire et hermitienne ] [ Nombre commentaires: 0] [nombre d'éditeurs: 1 ] [Editeur(s): Emmanuel Vieillard-Baron ] [nombre d'auteurs: 1 ] [Auteur(s): Patrice Lassère ]
Accordéon
Titre
Solution
Texte

Étude de \(\varphi\,:\,A\in M_n(\mathbb R)\longmapsto \varphi(A)=-A+\text{tr}(A)I_n.\)
Par Patrice Lassère le 7 novembre 2022 22:15

\(\varphi\) est bien entendu élément de \(\mathscr L(M_n(\mathbb R))\). Soit \(\lambda\) une valeur propre de \(\varphi\) : il existe une matrice \(A\) non nulle telle que

\[\varphi(A)= -A+\text{tr}(A)I_n=\lambda A\]

si bien que

\[(1+\lambda)A=\text{tr}(A)I_n \qquad\qquad(\bigstar)\]

-Si la trace de \(A\) est nulle, \(\lambda\) est forcément (\(A\) est non nulle) égal à \(-1\) et réciproquement, \(\lambda=-1\) implique \(A\) de trace nulle : \(\lambda=-1\) est bien valeur propre de \(\varphi\) et son sous-espace propre associé \(E_{-1}\) est le sous-espace constitué des matrices de trace nulle, classiquement de dimension \(n^2-1\) (considérer la forme linéaire \(A\mapsto\text{tr}(A)\) et appliquer le théorème du rang)

-Si la trace de \(A\) est non nulle, \(\lambda\) est différent de \(-1\) et \((\bigstar)\) nous donne

\[A=(1+\lambda)^{-1}\text{tr}(A)I_n \quad\Longrightarrow\quad \text{tr}(A)={n\text{tr}(A)\over (1+\lambda)}\quad\Longrightarrow\quad \lambda=n-1 \quad\&\quad A\in\textsf{vect}(I_n)\]

i.e. \(\lambda=n-1\) est aussi valeur propre, et le sous-espace propre associé est la droite vectorielle engendrée par \(I_n\) donc de dimension \(1\).

-En conclusion, \(\varphi\) admet deux valeurs propres \(-1\) et \(n-1\), les sous-espaces propres associés sont de dimensions respectives \(n^2-1\) et \(1\) donc de somme \(n^2=\text{dim}M_n(\mathbb R)\) et \(\varphi\) est diagonalisable (et son polynôme caractéristique \(P_{\varphi}(x)=(-1)^{n^2}(x+1)^{n^2-1}(x+1-n)\)...).


Espaces vectoriels, dimension, réduction des endomorphismes *

7 novembre 2022 22:15 — Par Patrice Lassère

(
[rms], 1999/2000). Soient \(A,B,C\in M_2(\mathbb R)\). Montrer qu’il existe \((a,b,c)\in\mathbb R^3\setminus\{(0,0,0)\}\) tel que \(aA+bB+cC\) possède une valeur propre double.



[ID: 2315] [Date de publication: 7 novembre 2022 22:15] [Catégorie(s): Algèbres bilinéaire et hermitienne ] [ Nombre commentaires: 0] [nombre d'éditeurs: 1 ] [Editeur(s): Emmanuel Vieillard-Baron ] [nombre d'auteurs: 1 ] [Auteur(s): Patrice Lassère ]
Accordéon
Titre
Solution
Texte

Espaces vectoriels, dimension, réduction des endomorphismes
Par Patrice Lassère le 7 novembre 2022 22:15

Dans l’espace vectoriel \(M_2(\mathbb R)\) de dimension \(4\) il n’y a qu’une alternative : ou bien la famille \(\{A,B,C\}\) est liée et il n’y a rien à démontrer, ou bien elle est libre et dans ce cas l’ensemble \[\textsf{vect}\{A,B,C\}=\{aA+bB+cC,\ (a,b,c)\in\mathbb R^3\}\] sous-espace vectoriel de dimension 3 est alors tenu, pour des raisons évidentes de dimension, de rencontrer le sous-espace \[\left\{\begin{pmatrix} a & b \\ 0 & a\end{pmatrix} , (a,b)\in\mathbb R^2\right\}\] de dimension \(2\) constitué de matrices à valeurs propres doubles.Le résultat suit.


Rayon spectral et décomposition de Dunford *

7 novembre 2022 22:15 — Par Patrice Lassère

En utilisant la décomposition de Dunford, montrer que pour toute matrice \(A\in M_n(\mathbb C)\) on a : \[\rho(A)=\lim_{k\to\infty}\Vert A^k\Vert^{1\over k}\]



[ID: 2317] [Date de publication: 7 novembre 2022 22:15] [Catégorie(s): Algèbres bilinéaire et hermitienne ] [ Nombre commentaires: 0] [nombre d'éditeurs: 1 ] [Editeur(s): Emmanuel Vieillard-Baron ] [nombre d'auteurs: 1 ] [Auteur(s): Patrice Lassère ]
Accordéon
Titre
Solution
Texte

Rayon spectral et décomposition de Dunford
Par Patrice Lassère le 7 novembre 2022 22:15

Par Dunford \(A=D+N\) avec \(D\) diagonalisable de mêmes valeurs propres que \(A\), \(N\) nilpotente et \(ND=DN\). donc pour \(k\geq n\) on aura \(N^k=0\) et

\[A^k=(D+N)^k=\sum_{l=0}^kC_n^lD^{k-l}N^l=\sum_{l=0}^nC_n^lD^{k- l}N^l=D^{k-n}\sum_{l=0}^nC_n^lD^{n-l}N^l.\]

Soit, en posant \(\alpha=\sup_{0\leq l\leq n} \{\Vert D\Vert^{n-l}\Vert N\Vert^l\}\)

\[\Vert A^k\Vert\leq \Vert D\Vert^{k-n}\sum_{l=0}^nC_k^l \Vert D\Vert^{n-l}\Vert N\Vert^l\leq \alpha\Vert D\Vert^{k-n}\left(\sum_{l=0}^nC_k^l\right)\]

or, pour \(0\leq l\leq n\) et \(k\geq n\)

\[C_k^l\leq k(k-1)\dots(k-l+1)\leq k^l\leq k^n\]

si bien que

\[\Vert A^k\Vert\leq \alpha (n+1)k^n\Vert D^{k-n}\Vert\]

et

\[\rho(A)\leq\Vert A^k\Vert^{1\over k}\leq \left(\alpha(n+1)\right)^{1\over k} k^{n\over k}\Vert D^{k-n}\Vert^{1\over k},{(\bigstar)}\]

en remarquant enfin que \(\displaystyle\lim_{k\to\infty}\left(\alpha(n+1)\right)^{1\over k} k^{n\over k}=1\) et

\[\lim_{k\to\infty}\Vert D^{k-n}\Vert^{1\over k} =\lim_{k\to\infty}\left(\Vert D^{k-n}\Vert^{1\over k-n}\right)^{k-n\over k} =\lim_{k\to\infty} \Vert D^{k-n}\Vert^{1\over k-n} =\rho(D) =\rho(A)\]

avec \((\bigstar)\), il vient finalement \(\displaystyle\rho(A)=\lim_{k\to\infty}\Vert A^k\Vert^{1\over k}\).


Matrices nilpotentes *

7 novembre 2022 22:15 — Par Patrice Lassère

Existe-t-il une matrice \(A\in M_n(\mathbb R)\) nilpotente à coefficients \(>0\) ?



[ID: 2319] [Date de publication: 7 novembre 2022 22:15] [Catégorie(s): Algèbres bilinéaire et hermitienne ] [ Nombre commentaires: 0] [nombre d'éditeurs: 1 ] [Editeur(s): Emmanuel Vieillard-Baron ] [nombre d'auteurs: 1 ] [Auteur(s): Patrice Lassère ]
Accordéon
Titre
Solution
Texte

Matrices nilpotentes
Par Patrice Lassère le 7 novembre 2022 22:15

Si une telle matrice existe la matrice \(A^k\) est encore à coefficients \(>0\) pour tout entier \(k\in\mathbb N\) ; mais \(A\) nilpotente implique (Cayley-Hamilton par exemple) \(A^n=0\), contradiction.


Matrice, comatrice et rang *

7 novembre 2022 22:38 — Par Patrice Lassère

Soit \(A\in M_d(\mathbb C)\), étudier le rang de la comatrice de \(A\) en fonction du rang de \(A\).



[ID: 2321] [Date de publication: 7 novembre 2022 22:38] [Catégorie(s): Algèbres bilinéaire et hermitienne ] [ Nombre commentaires: 0] [nombre d'éditeurs: 1 ] [Editeur(s): Emmanuel Vieillard-Baron ] [nombre d'auteurs: 1 ] [Auteur(s): Patrice Lassère ]
Accordéon
Titre
Solution
Texte

Matrice, comatrice et rang
Par Patrice Lassère le 7 novembre 2022 22:38

L’identité suivante est toujours vérifiée dans \(M_d(\mathbb C)\)

\[^t\!(\text{com}(A))A=A\,^t\!(\text{com}(A))=\det(A)I_d.\] et on a aussitôt : \[\text{rang}(A)=d \quad\iff\quad \text{rang}(\,^t\!(\text{com}(A)))=d,\] dans le cas contraire le déterminant de \(A\) est nul et notre formule devient \[^t\!(\text{com}(A))A=A\,^t\!(\text{com}(A))=0\] soit, identifiant canoniquement matrice et endomorphisme

\[\left(\text{im}(^t\!(\text{com}(A)))\subset\ker(A)\right)\quad \&\quad \left(\text{im}(A)\subset\ker(^t\!(\text{com}(A)))\right)\] et par suite

\[\forall\,A\in M_d(\mathbb C)\quad\text{rang}^t\!(\text{com}(A))+\text{rang}(A)\leq d\qquad\qquad(\text{$\star$})\] \(\rightsquigarrow\quad\) Si \(\text{rang}(A)=d-1\) la matrice \(A\) admet un mineur d’ordre \(d-1\) non nul et donc la comatrice de \(A\) admet un coefficient non nul, son rang est donc supérieur ou égal à \(1\) et égal à \(1\), vu \((\text{$\star$})\). De même \(\det( A)=0\) et \(\text{rang}^t\!(\text{com}(A))\geq 1\) exigent \(\text{rang}^t\!(\text{com}(A))=1\) et \(\text{rang}(A)=d-1\).

\(\rightsquigarrow\quad\) Si \(\text{rang}(A)\leq d-2\) tous les mineurs d’ordre \(d-1\) de \(A\) sont nuls et par suite la comatrice de \(A\) est la matrice nulle, donc de rang nul et réciproquement. Résumons nous

\[\begin{cases} \text{rang}(A)=d &\quad\iff\quad \text{rang}^t\!(\text{com}(A))=d,\\ \text{rang}(A)=d-1 &\quad\iff\quad \text{rang}^t\!(\text{com}(A))=1\quad{pour }\ d\geq 2,\\ \text{rang}(A)\leq d-2 &\quad\iff\quad \text{rang}^t\!(\text{com}(A))=0. \end{cases}\]


Séries entières, algèbre linéaire *

7 novembre 2022 22:38 — Par Patrice Lassère

Pour \(A\in M_d(\mathbb C)\), quel est le rayon de convergence de la série entière \(f_A(z):=\sum_{k\geq 0}\text{tr}(A^k)z^k\) ?

Exprimer \(f_A\) en fonction du polynôme caractéristique et de ses dérivées.



[ID: 2323] [Date de publication: 7 novembre 2022 22:38] [Catégorie(s): Algèbres bilinéaire et hermitienne ] [ Nombre commentaires: 0] [nombre d'éditeurs: 1 ] [Editeur(s): Emmanuel Vieillard-Baron ] [nombre d'auteurs: 1 ] [Auteur(s): Patrice Lassère ]
Accordéon
Titre
Solution
Texte

Séries entières, algèbre linéaire
Par Patrice Lassère le 7 novembre 2022 22:38

Notons \(\lambda_1,\dots,\lambda_d\) les valeurs propres de \(A\) comptées avec leur multiplicités. \(A\) est diagonalisable dans \(\mathbb C\) et pour tout entier \(k\)

\[\text{tr}(A^k)=\sum_{j=0}^d\lambda_j^k\] par conséquent, si les \(\lambda_j\) ne sont pas tous non nuls (i.e. \(A\) n’est pas nilpotente) le rayon de convergence de notre série entière est supérieur ou égal (et en fait égal) à

\[\rho:=\inf_{\lambda_j\ne 0,\,1\leq j\leq d}\ \vert\lambda_j\vert^{-1}.\]

Pour \(\vert z\vert<\rho\)

\[\begin{aligned}f(z)=\sum_{k\geq 0}\text{tr}(A^k)z^k &=\sum_{k\geq 0}\lambda_1^kz^k+\dots+\sum_{k\geq 0}\lambda_d^kz^k\\ &={1\over{1-\lambda_1z}}+\dots+{1\over{1-\lambda_dz}}\\ &={{\chi'_A(z^{-1})}\over{z\chi_A(z^{-1})}}\qquad(\bigstar)\\ \end{aligned}\]

Enfin, si \(A\) est nilpotente \(f\equiv d\) puisque \(\text{tr}(A^k)=0\,,\,\forall k\in\mathbb N^\star\) et \(\text{tr}(A^0=I_d)=d\). Dans ce dernier cas, il faut remarquer que la formule \((\bigstar)\) subsiste encore puisque \(\chi_A(z)=(-1)^dz^d\).


Matrices semblables, polynômes *

7 novembre 2022 22:38 — Par Patrice Lassère

Montrer que deux matrices réelles \(A,B\in M_n(\mathbb R)\) semblables dans \(M_n(\mathbb C)\) sont encore semblables dans \(M_n(\mathbb R)\).



[ID: 2325] [Date de publication: 7 novembre 2022 22:38] [Catégorie(s): Algèbres bilinéaire et hermitienne ] [ Nombre commentaires: 0] [nombre d'éditeurs: 1 ] [Editeur(s): Emmanuel Vieillard-Baron ] [nombre d'auteurs: 1 ] [Auteur(s): Patrice Lassère ]
Accordéon
Titre
Solution
Texte

Matrices semblables, polynômes
Par Patrice Lassère le 7 novembre 2022 22:38

Il existe donc \(P\in GL_n(\mathbb C)\) telle que \(A=P^{-1}B P\), soit si \(P_1=\text{re}(P)\) et \(P_2=\text{im}(P) : (P_1+iP_2)A=B(P_1+iP_2)\) i.e. prenant parties réelles et imaginaires : \[\,P_1A=BP_1\quad \&\quad P_2A=BP_2.{(\bigstar)}\] On considère alors l’application : \(\varphi\,:\,z\in\mathbb C\mapsto\,\varphi(z)=\text{det}(P_1+zP_2)\). \(\varphi\in\mathbb C[z]\) et n’est pas identiquement nulle car \(\varphi(i)=\text{det}(P)\) : il existe donc (un polynôme non nul ne possède dans \(\mathbb C\) qu’un nombre fini de racines) \(x\in\mathbb R\) tel que \(\varphi(x)\in\mathbb R^\star\), autrement dit \(Q=P_1+xP_2\in\,GL_n(\mathbb R)\) et avec \((\bigstar)\) on a immédiatement \(A=Q^{-1}BQ\)

Remarque : c’est un cas particulier du résultat suivant :

Soit \(\mathbb L\) une extension du corps \(\mathbb K\). Soit \(M\) et \(N\in M_n(\mathbb K)\) deux matrices semblables dans \(M_n(\mathbb L)\), alors elles sont semblables dans \(M_n(\mathbb K)\) .


Réduction des endomorphismes *

7 novembre 2022 22:38 — Par Patrice Lassère

Soit \(A\in M_2(\mathbb Z)\), s’il existe \(N\in\mathbb N\) tel que \(A^N=I_2\), montrer que \(A^{12}=I_2\).



[ID: 2327] [Date de publication: 7 novembre 2022 22:38] [Catégorie(s): Algèbres bilinéaire et hermitienne ] [ Nombre commentaires: 0] [nombre d'éditeurs: 1 ] [Editeur(s): Emmanuel Vieillard-Baron ] [nombre d'auteurs: 1 ] [Auteur(s): Patrice Lassère ]
Accordéon
Titre
Solution
Texte

Réduction des endomorphismes
Par Patrice Lassère le 7 novembre 2022 22:38

\(A\) est annulée par \(P(X)=X^N-1\) scindé à racines simples, elle est donc diagonalisable sur \(\mathbb C\). Ses deux valeurs propres \(\lambda_1,\lambda_2\) sont racines de \(P\) donc des racines \(N\)-ièmes de l’unité. Enfin, \(A\) étant à coefficients réels, si elle admet une valeur propre non réelle : les deux seront conjuguées (\(\lambda_1=\overline{\lambda_2}\))

\(\rightsquigarrow\)Si les valeurs propres sont réelles \(\lambda_1,\lambda_2\in\{-1,+1\}\) et \(A\) étant diagonalisable \(A^2=I_2\).

\(\rightsquigarrow\)Sinon, elles sont conjuguées, mais \(A\) étant à coefficients dans \(\mathbb Z\) implique

\[\lambda_1+\lambda_2 =2\text{re}(\lambda_1)\in\mathbb Z\Longrightarrow 2\text{re}(\lambda_1)\in\{-2,-1,0,1,2\}.\]

On en déduit facilement que \(\lambda_1\) est racine seconde, troisième ou quatrième de l’unité, dans tous les cas \(A^{12}=I_2\).

Remarque : En résumé, l’ordre d’une matrice \(A\in GL_n(\mathbb Z)\) appartient à \(\left\lbrace 1,2,3,4,6,+\infty\right\rbrace\) et ces valeurs sont atteintes. Par exemple l’ordre des matrices ci-dessous \[\begin{pmatrix} 1&0\\0&1\end{pmatrix}, \begin{pmatrix} 0&1\\1&0\end{pmatrix}, \begin{pmatrix} 0&-1\\1&-1\end{pmatrix}, \begin{pmatrix} 0&-1\\1&0\end{pmatrix}, \begin{pmatrix} 0&-1\\1&1\end{pmatrix}, \begin{pmatrix} 1&1\\0&1\end{pmatrix}\] est respectivement \(1,2,3,4,6\) et \(+\infty\).


Réduction des endomorphismes *

7 novembre 2022 22:38 — Par Patrice Lassère

Soit \(A\in M_3(\mathbb R)\) vérifiant \(A^4=A^2\). Si \(1\) et \(-1\) sont valeurs propres de \(A\), montrer que \(A\) est diagonalisable.



[ID: 2329] [Date de publication: 7 novembre 2022 22:38] [Catégorie(s): Algèbres bilinéaire et hermitienne ] [ Nombre commentaires: 0] [nombre d'éditeurs: 1 ] [Editeur(s): Emmanuel Vieillard-Baron ] [nombre d'auteurs: 1 ] [Auteur(s): Patrice Lassère ]
Accordéon
Titre
Solution
Texte

Réduction des endomorphismes
Par Patrice Lassère le 7 novembre 2022 22:38

Le polynôme \(X^4-X^2=X^2(X-1)(X+1)\) est annulé par \(A\), le spectre de \(A\) est donc inclus dans \(\{-1,0,1\}\) et vu les hypothèses nous avons donc deux alternatives :

\(\text{Sp}(A)=\{-1,0,1\}\) et \(P_A(X)=-X(X+1)(X-1)\) est scindé à racines simples : \(A\) est diagonalisable.

\(0\) n’est pas valeur propre de \(A\) : dans ce cas \(A\in GL_3(\mathbb R)\) et alors \[A^4=A^2\Longrightarrow A^2=I_2,\] \(A\) annulée par un polynôme scindé à racines simples est encore diagonalisable (ce dernier cas correspond au cas sp\((A)=\{-1,1\}\) et donc \(P_A(X)=(X-1)^2(X+1)\text{ ou } (X-1)(X+1)^2.\)


Groupes, réduction des endomorphismes *

7 novembre 2022 22:38 — Par Patrice Lassère

Soient \(n>p\) deux entiers. Montrer que les groupes \(GL_n(\mathbb K)\) et \(GL_p(\mathbb K)\) ne sont pas isomorphes.



[ID: 2331] [Date de publication: 7 novembre 2022 22:38] [Catégorie(s): Algèbres bilinéaire et hermitienne ] [ Nombre commentaires: 0] [nombre d'éditeurs: 1 ] [Editeur(s): Emmanuel Vieillard-Baron ] [nombre d'auteurs: 1 ] [Auteur(s): Patrice Lassère ]
Accordéon
Titre
Solution
Texte

Groupes, réduction des endomorphismes
Par Patrice Lassère le 7 novembre 2022 22:38

Pour cela considérons \[G:=\left\{\,A\in GL_n(\mathbb K)\text{ diagonales et telles que sp}(A)\subset\{-1,1\}\right\},\] c’est un sous-groupe commutatif de cardinal \(2^n\) de \(GL_n(\mathbb K)\). Supposons alors qu’il existe un isomorphisme \(\varphi\,:\,GL_n(\mathbb K)\to GL_p(\mathbb K)\). Alors \(\varphi(G)=H\) est un sous-groupe commutatif de \(GL_p(\mathbb K)\) de cardinal \(2^n\), dont les éléments sont annulés par \(X^2-1\) : ils sont donc diagonalisables à valeurs propres dans \(\{-1,+1\}\). \(\varphi(G)\) étant commutatif il est alors bien connu ([deswar]-2, prop. 55 page 220) que ses éléments sont simultanément diagonalisables i.e. \[\exists P\in GL_p(\mathbb K)\ \text{ telle que }\,\forall\,B\in\varphi(G)\ :\,P^{-1}BP\text{ est diagonale.}\] \(P^{-1}BP\) est alors diagonale à spectre dans \(\{-1,+1\}\) ce qui implique que \(2^n=\vert\varphi(G)\vert\leq 2^p\) inégalité absurde puisque \(n>p\).


Inégalité, matrices, déterminant *

7 novembre 2022 22:38 — Par Patrice Lassère

Soient \(c\in\mathbb R_+,\ A=((a_{i,j}))\in M_n(\mathbb C)\) vérifiant \(\vert a_{i,j}\vert\leq c\) pour tous \(1\leq i,j\leq n\). Montrer que \[\vert\text{det}(A)\vert\leq c^n n^{n/2}.\]



[ID: 2333] [Date de publication: 7 novembre 2022 22:38] [Catégorie(s): Algèbres bilinéaire et hermitienne ] [ Nombre commentaires: 0] [nombre d'éditeurs: 1 ] [Editeur(s): Emmanuel Vieillard-Baron ] [nombre d'auteurs: 1 ] [Auteur(s): Patrice Lassère ]
Accordéon
Titre
Solution
Texte

Inégalité, matrices, déterminant
Par Patrice Lassère le 7 novembre 2022 22:38

Seul le cas où \(A\in GL_n(\mathbb C)\) mérite explication. Pour une telle matrice désignons par \(C_1,\dots,C_n\) ses colonnes.

-Supposons les colonnes \((C_i)_1^n\) deux à deux orthogonales. Si \[D_i=\Vert C_i\Vert^{-1}C_i,\quad\text{et}\quad B=[D_1,\dots,D_n]\] la matrice \(B\) est orthogonale donc \(\vert\text{det}(B)\vert=1\) ; vu que \[\forall\,1\leq j\leq n\ :\ \Vert C_j\Vert=\sqrt{a_{1,j}^2+\dots+a_{n,j}^2}\leq c\sqrt{n}\] on a \[\vert \text{det}(A)\vert=\vert{det}(B)\vert\,\Vert C_1\Vert\dots\Vert C_n\Vert \leq c^n n^{n/2}.{\text{($\star$)}}\]

-Pour le cas général, appliquons le procédé d’orthonormalisation de Schmidt à la famille libre \((C_i)_1^n\) : soit \(D_1=C_1\), et \(D_2=C_2+\lambda_{2,1}C_1\) où comme toujours la constante \(\lambda_{2,1}\) est déterminée de sorte que \(D_2\) soit orthogonal à \(D_1\) i.e. \[\langle C_2,D_1\rangle+\lambda_{2,1}^2\Vert D_1\Vert^2 =0\] avec Pythagore \[\Vert D_2\Vert =\Vert C_2\Vert^2-\lambda_{2,1}^2\Vert D_1\Vert^2\leq\Vert C_2\Vert^2.\] De même \[D_k=C_k-\lambda_{k,1}D_1+\dots+\lambda_{k,k-1}D_{k-1}\] et avec \[\langle C_k,D_j\rangle+\lambda_{k,j}^2\Vert D_j\Vert^2 =0,\quad\forall\,1\leq j\leq k-1\] on a \[\begin{aligned} \Vert D_k\Vert &=\Vert C_k\Vert^2+\sum_{j=1}^{k-1}\lambda_{k,j}^2\Vert D_j\Vert^2 -2\sum_{j=1}^{k-1}\lambda_{k,j}^2\Vert D_j\Vert^2\\ &\leq \Vert C_k\Vert^2-\sum_{j=1}^{k-1}\lambda_{k,j}^2\Vert D_j\Vert^2 \leq\vert C_k\Vert^2 \end{aligned}\] soit \[\Vert D_k\Vert\leq \Vert C_k\Vert,\quad\forall\,1\leq k\leq n{\text{($\star$)}}\] puisque bien entendu \(\text{det}[C_1,\dots,C_n]=\text{det}[D_1,\dots,D_n]\) ; l’inégalité est démontrée vu (\(\star\)) et (\(\star\)).


Un théorème de Kronecker *

7 novembre 2022 22:38 — Par Patrice Lassère

Par polynôme de Sylvester on entend tout polynôme \(P\in {\Bbb Z}[X]\) unitaire à racines de module inférieur où égal à \(1\). Montrer que

(Kronecker) Les zéros non nuls d’un polynôme de Sylvester sont des racines de l’unité.



[ID: 2335] [Date de publication: 7 novembre 2022 22:38] [Catégorie(s): Algèbres bilinéaire et hermitienne ] [ Nombre commentaires: 0] [nombre d'éditeurs: 1 ] [Editeur(s): Emmanuel Vieillard-Baron ] [nombre d'auteurs: 1 ] [Auteur(s): Patrice Lassère ]
Accordéon
Titre
Solution
Texte

Un théorème de Kronecker
Par Patrice Lassère le 7 novembre 2022 22:38

Soit \(n\in {\Bbb N}^\star\). Notons \(Z_n\) la collection des zéros (comptés avec leurs multiplicités) de l’ensemble \(\mathscr S_n\) des polynômes de Sylvester de degré inférieur où égal à \(n\).

- Étape 1 : Pour \(n\in {\Bbb N}^\star\) : \(s_n:=\text{card}(\mathscr S_n)\) est fini.

Soit \(p(z)=z^n+a_{n-1}z^{n-1}+\dots +a_0\,=\, \prod_{i=1}^n(z-\zeta_i)\,\in \mathscr S_n\), avec les formules de Newton, si \(\ 0\leq k\leq n-1\) on a \[\vert a_k\vert =\Big\vert \sum_{1\leq i_1<\dots<i_k\leq n}(-1)^k \zeta_{i_1} \dots \zeta_{i_k}\Big\vert \leq \sum_{1\leq i_1<\dots<i_k\leq n} \vert \zeta_{i_1} \dots \zeta_{i_k} \vert \leq {\textsf C}_n^k\leq n!,\] les coefficients \(a_k\) étant entiers, il n’y a qu’un choix fini de \(a_k\) et \(s_n\) est fini.

- Étape 2 : \(\left(\ \zeta\in Z_n\right) \Longrightarrow \left(\ \zeta^k\in Z_n,\ \forall\,k\in \mathbb N\right)\).

Soit \(p(z)=z^n+a_{n-1}z^{n-1}+\dots +a_0 \in \mathbb Z[X]\), sa matrice compagnon \[C_p =\ \begin{pmatrix} 0 & 0 & \dots & 0 & -a_0 \\ 1 & 0 & \dots & 0 & -a_1 \\ 0 & 1 & \dots & 0 & -a_2 \\ \vdots & \ddots & \ddots & \vdots & \vdots \\ 0 & \dots & 0 & 1 & -a_{n-1} \end{pmatrix}\in M_n(\Bbb Z)\] est triangularisable dans \(M_n(\Bbb C)\), il existe \(G\in GL_n(\Bbb C)\) telle que \[C_p=G^{-1} \begin{pmatrix} \zeta_1 & & \text{\ \Huge ?}\\ &\ddots & \\ \text{\ \Huge 0} & &\zeta_n \end{pmatrix}G.\] Mais alors \[C_p^N=G^{-1} \begin{pmatrix} \zeta_1^N & & \text{\ \Huge ?}\\ &\ddots & \\ \text{\ \Huge 0} & &\zeta_n^N \end{pmatrix}G\in M_n(\Bbb Z),\] autrement dit \(C_p^N\) est une matrice à coefficients dans \(\Bbb Z\) qui admet \(\zeta_1^N,\dots \zeta_n^N\) comme valeurs propres : son polynôme caractéristique répond à la question.

- Étape 3 : La conclusion.

Supposons qu’il existe un polynôme \(p\in K_n\) admettant au moins une racine, (disons \(\zeta_1\)) qui ne soit ni racine de l’unité, ni de module strictement compris entre \(0\) et \(1\), l’ensemble \(\{ \zeta_1^N\}_{N\in\Bbb N^\star}\) est alors de cardinal infini. D’autre part, par la seconde étape \(p_{C_p^N}(\zeta_1^N)=0,\ \forall N\in\Bbb N^\star\), si bien que l’ensemble infini \(\{ \zeta_1^N\}_{N\in\Bbb N^\star}\) est inclu dans \(Z_n\) de cardinal fini (étape \(1\)) et on a la contradiction désirée.

Remarques : -On peut tout aussi bien montrer qu’un polynôme de Sylvester est sans zéros de module strictement compris entre \(0\) et \(1\) de la manière suivante : soit \(p(z) = z^k\prod_{i=k+1}^n(z-\zeta_i)\in K_n\), toujours avec Newton : \(1\leq \vert a_k\vert = \vert \zeta_{k+1}\dots \zeta_n \vert \leq 1,\) soit \(\vert \zeta_{k+1}\vert =\dots = \vert \zeta_n\vert =1.\)

-On pourra aussi consulter les ouvrages de J.M.Arnaudies & J.Bertin "Groupes, Algèbre et Géométrie" tome 1, pages 127-128, Ellipse, (1993) ou E.Leichnam "Exercices corrigés de Mathématiques, Polytechnique, ENS" (Algèbre et Géométrie), exercice 1-30, Ellipse, (1999) pour d’autres approches.

-Un entier algébrique est une racine d’un polynôme unitaire \(P(X)=X^d+a_{d-1}X^{d-1}+\dots+a_1X+a_0\in\mathbb Z[X]\). On a

Un entier algébrique est soit entier, soit irrationnel.

Cette assertion repose essentiellement sur les idées précédentes, en voici donc les étapes principales :

\(\rightsquigarrow\quad\) \(\alpha\in\mathbb C\) est un entier algébrique si, et seulement si, il est valeur propre d’une matrice à coefficients entiers.

\(\rightsquigarrow\quad\) Si \(A\in M_n(\mathbb Z)\) admet une valeur propre \(\lambda\) rationnelle, alors \(A\) admet un vecteur propre à coordonnées entières associé à la valeur propre \(\lambda\).

\(\rightsquigarrow\quad\) Soit \(\alpha\) un entier algébrique racine d’un polynôme \(P\in\mathbb Z[X]\) de degré \(n\geq 2\). Si \(\alpha\in\mathbb Q\setminus\mathbb Z\) considérons \(k\in\mathbb Z\) tel que \(k<\alpha<k+1\) et \(A:=C_P-kI_n\in M_n(\mathbb Z)\). On vérifie facilement que \(\lambda:=\alpha-k\) est valeur propre de \(A\), puis en considérant la suite \((A^mX)_m\)\(X\in\mathbb Z^n\) vérifie \(AX=\lambda X\), conclure.


Le commutant dans \(M_2(\mathbb K)\) *

7 novembre 2022 22:38 — Par Patrice Lassère

À toute matrice \(A\in M_2(\mathbb K)\) on associe \[\mathscr C_A:=\left\lbrace B\in M_2(\mathbb K)\ :\ AB=BA\right\rbrace\] son commutant. Montrer que \(\mathscr C_A\) est de dimension \(2\) ou \(4\).



[ID: 2337] [Date de publication: 7 novembre 2022 22:38] [Catégorie(s): Algèbres bilinéaire et hermitienne ] [ Nombre commentaires: 0] [nombre d'éditeurs: 1 ] [Editeur(s): Emmanuel Vieillard-Baron ] [nombre d'auteurs: 1 ] [Auteur(s): Patrice Lassère ]
Accordéon
Titre
Solution
Texte

Le commutant dans \(M_2(\mathbb K)\)
Par Patrice Lassère le 7 novembre 2022 22:38

Soit \[A=aE_{11}+bE_{12}+cE_{21}+dE_{22}{(\bigstar)}\] la décomposition de \(A\) dans la base canonique de \(M_2(\mathbb K)\).

-Si \(A\) est scalaire (i.e. \(A=\lambda I_2\) ), \(\mathscr C_A=M_2(\mathbb K)\) est donc dimension \(4\).

-Si \(A\) est diagonale mais non scalaire, on vérifie encore sans peine avec \((\bigstar)\) que \(\mathscr C_A\) est de dimension \(2\).

-Sinon, \(A\) est non scalaire et \(A\) et \(I_2\) sont libres, et \(\mathscr C_A\) est de dimension supérieure ou égale à \(2\). Supposons \(\mathscr C_A\) de dimension supérieure ou égale à \(3\). Dans \(M_2(\mathbb K)\) de dimension \(4\) il se doit de rencontrer \[\mathscr E:=\mathbb K E_{11}+\mathbb K E_{12}.\] Soit donc \[B=\alpha E_{11}+\beta E_{12}\in \mathscr E\cap\mathscr C_A,\quad (\alpha,\beta)\neq(0,0)\] en écrivant \(AB=BA\) il vient \(c=0\). De même, en considérant \(\mathscr E:=\mathbb K E_{21}+\mathbb K E_{22}\) il vient \(b=0\) soit \(A\) diagonale ce qui est absurde donc \(\text{dim}\,(\mathscr C_A)< 3\), soit \(\text{dim}\,(\mathscr C_A)=2\).

Remarques : -Pour une matrice \(A\in M_n(\mathbb K)\), la dimension du commutant de \(A\) vérifie \[n\leq \text{dim}\,(\mathscr C_A) \leq n^2\] et la dimension vaut \(n\) si, et seulement si \(A\) est cyclique (i.e. \(A\) est semblable à une matrice compagnon ou encore polynômes minimaux et caractéristiques coïncident) et \(n^2\) si et seulement si \(A\) est semblable à la matrice identité.

-La question se pose alors de savoir si la dimension du commutant peut prendre toutes les valeurs comprises entre \(n\) et \(n^2\). L’exercice que nous venons de traiter montre que la réponse à cette question est non si \(n=2\) car \(\text{dim}\,\mathscr C_A\) à priori dans \(\{2,3,4\}\) ne peut jamais être égale à \(3\). L’explication de ce phénomène est résumée dans le résultat qui suit

Car pour \(n=2\), l’entier \(3\) est le seul élément de \(\{2,3,4\}\) qui ne vérifie pas ces deux propriétés. Ce dernier résultat est délicat à établir 1 ; pour notre exercice, il est d’ailleurs plus rapide (([rms], 2000/01, ex. 15)) d’établir (c’est d’ailleurs un corollaire immédiat du précédent) que la codimension du commutant est toujours paire ce qui n’est pas le cas de l’entier \(3\) (\(4-3=1\)...).


  1. voir Carrieu, Fadel, Fieux, Lassère, Rodriguez Autour des matrices de Frobenius ou Compagnon↩︎


Points isolés des solutions de l’équation \(X^2=I_n\) dans \(M_n(\mathbb R)\) * Polytechnique

7 novembre 2022 22:38 — Par Patrice Lassère

Dans \({\mathscr M}_n ( {\mathbb{R}} )\), quel est l’ensemble des points isolés de l’ensemble des matrices dont le carré est égal à \(I_n\) ?



[ID: 2339] [Date de publication: 7 novembre 2022 22:38] [Catégorie(s): Algèbres bilinéaire et hermitienne ] [ Nombre commentaires: 0] [nombre d'éditeurs: 1 ] [Editeur(s): Emmanuel Vieillard-Baron ] [nombre d'auteurs: 1 ] [Auteur(s): Patrice Lassère ]
Accordéon
Titre
Solution
Texte

Points isolés des solutions de l’équation \(X^2=I_n\) dans \(M_n(\mathbb R)\)
Par Patrice Lassère le 7 novembre 2022 22:38

Posons \(A=\{M\in {\mathscr M}_n(\mathbb{R})\; \vert\; M^2=I_n\}\). Nous allons montrer que les points isolés de \(A\) sont \(I_n\) et \(-I_n\). Le cas \(n=1\) étant trivial, on suppose dans la suite \(n\geqslant 2\).

Soit \(M\in A\). Le polynôme \(X^2-1=(X-1)(X+1)\) étant annulateur de \(M\), la matrice \(M\) est semblable à une matrice \(\begin{pmatrix}I_p & 0\\ 0& -I_{n-p}\end{pmatrix}\) avec \(0\leqslant p\leqslant n\). On a \[\mathop{\rm tr} M=2p-n,\quad \mathop{\rm tr}M=n\Leftrightarrow M=I_n\quad \text{ et }\quad\mathop{\rm tr}M =-n \Leftrightarrow M=- I_n.\]

Soit \(\varphi\) l’application de \(A\) dans \(\mathbb{R}\) : \(M\mapsto \mathop{\rm tr}M\). L’application \(\varphi\) est continue et \(\{I_n\} =\varphi^{-1}\big(]n-\frac{1}{2},n+\frac{1}{2}[\big)\). Ainsi, \(\{I_n\}\) est un ouvert de \(A\) et \(I_n\) est donc un point isolé de \(A\). On montre de même que \(-I_n\) est un point isolé de \(A\).

Soit \(M\in \, A\setminus \{I_n,-I_n\}\) : \(M=P\begin{pmatrix}I_p & 0\\ 0& -I_{n-p}\end{pmatrix}P^{-1}\), où \(P\in \mathop{\rm GL}_n(\mathbb{R})\) et \(1\leqslant p\leqslant n-1\). Notons \(E_{1,n}\) la matrice élémentaire dont tous les coefficients sont nuls sauf celui de la 1ère ligne et \(n\)-ème colonne qui vaut 1. On constate que \(M_k=P\left[\begin{pmatrix}I_p & 0\\ 0& -I_{n-p}\end{pmatrix}+\frac{1}{k}E_{1,n}\right]P^{-1}\) ( \(k\in\mathbb{N}^*\)) est élément de \(A\) et que la suite \((M_k)\) tend vers \(M\) sans prendre la valeur \(M\). Ainsi, \(M\) n’est pas isolé.

Remarque : Pour \(0\leqslant p\leqslant n\), on note \(A_p=\{M\in A\; \vert\; \mathop{\rm tr}M =2p-n\}\). On montre que les \(A_p\) sont les composantes connexes par arcs de \(A\). On utilise pour cela l’écriture \(M=P\begin{pmatrix}I_p & 0\\ 0& -I_{n-p}\end{pmatrix}P^{-1}\) et on montre qu’on peut choisir \(P\) dans \(\mathop{\rm SL}_n(\mathbb{R})\). Comme \(\mathop{\rm SL}_n(\mathbb{R})\) est connexe par arcs, \(A_p\) l’est aussi.


Sur l’équation \(\displaystyle\sin(A)=B\) *

7 novembre 2022 22:38 — Par Patrice Lassère

Putnam, 1996.

Existe-t-il une matrice \(A\in M_2(\mathbb C)\) vérifiant : \[\sin(A):=\sum_{n=0}^\infty\dfrac{(-1)^n}{(2n+1)!}A^{2n+1}=\begin{pmatrix}1&2005\\0&1\end{pmatrix}\quad ?\]



[ID: 2341] [Date de publication: 7 novembre 2022 22:38] [Catégorie(s): Algèbres bilinéaire et hermitienne ] [ Nombre commentaires: 0] [nombre d'éditeurs: 1 ] [Editeur(s): Emmanuel Vieillard-Baron ] [nombre d'auteurs: 1 ] [Auteur(s): Patrice Lassère ]
Accordéon
Titre
Solution
Texte

Sur l’équation \(\displaystyle\sin(A)=B\)
Par Patrice Lassère le 7 novembre 2022 22:38

-Solution 1 : Supposons qu’une telle matrice existe.

\(\rightsquigarrow\quad\) Si les deux valeurs propres (\(\alpha\) et \(\beta\)) de \(A\) sont distinctes, \(A\) est diagonalisable ; il existe donc \(C\in GL_n(\mathbb C)\) telle que \[B=\begin{pmatrix}\alpha&0\\0&\beta\end{pmatrix}=CAC^{-1},\] on à alors (car \(A^n=CB^nB^{-1}\)) \[\sin(A)=C\sin(B)C^{-1}\quad\text{et}\quad \sin(B)= \sum_{n=0}^\infty\dfrac{(-1)^n}{(2n+1)!}\begin{pmatrix}\alpha^{2n+1}&0\\0&\beta^{2n+1}\end{pmatrix}^{2n+1}=\begin{pmatrix}\sin(\alpha)&0\\0&\sin(\beta)\end{pmatrix}\] \(\sin(A)\) est donc diagonalisable ce qui est contraire à l’hypothèse.

\(\rightsquigarrow\quad\) Envisageons maintenant les cas où les valeurs propres de \(A\) sont égales. Il existe \(C\in GL_n(\mathbb C)\) telle que \[B=\begin{pmatrix}x&y\\0&x\end{pmatrix}=CAC^{-1},\] soit (en calculant \(B^n\) pour tout \(n\in\mathbb N\)) \[\sin(B)=C\sin(A)C^{-1}=\begin{pmatrix}\sin(x)&y\cos(x)\\0&\sin(x)\end{pmatrix}\] mais \(1\) est l’unique valeur propre de \(A\), donc \(\sin(x)=0\), \(\cos(x)=0\), \(B=I_2\) et finalement \(A=I_2\) ce qui est absurde.

-Solution 2 : Avec \[\cos(A)=\sum_{n=0}^\infty \dfrac{(-1)^n}{(2n)!}A^{2n}\] on vérifie que \[\sin^2(A)+\cos^2(A)=I_2\] si bien que \[\sin(A)=\begin{pmatrix}1&2005\\0&1\end{pmatrix}\] implique \[\cos^2(A)=\begin{pmatrix}0&-2.2005\\0&0\end{pmatrix}.\] Cette dernière équation implique que la matrice \(\cos^2(A)\) est nilpotente, étant de taille \(2\times 2\) elle ne peut être que nulle ce qui est absurde.


Quelques propriétés topologiques de \(\mathscr O_n(\mathbb R)\) et \(\mathscr U_n(\mathbb C)\) *

7 novembre 2022 22:38 — Par Patrice Lassère

  1. Montrer que \(\mathscr O_n(\mathbb R)\) (resp. \(\mathscr U_n(\mathbb C)\)) est compact dans \(M_n(\mathbb R)\) (resp. \(M_n(\mathbb C)\)).

  2. Décomposition polaire généralisée : montrer que \[\forall\,M\in M_n(\mathbb R)\ :\ \exists\,(O,S)\in\mathscr O_n(\mathbb R)\times\mathscr S_n^+\ \text{ telles que } M=OS,\] \[\forall\,M\in M_n(\mathbb C)\ :\ \exists\,(U,H)\in\mathscr U_n(\mathbb C)\times\mathscr H_n^+\ \text{ telles que } M=UH.\]

  3. Montrer que \[GL_n(\mathbb R)\underset{\text{top.}}{\simeq} \mathscr O_n(\mathbb R)\times\mathscr S_n^{+\star}\quad\text{ et }\quad GL_n(\mathbb C)\underset{\text{top.}}{\simeq} \mathscr U_n(\mathbb R)\times\mathscr H_n^{+\star}.\]

  4. Montrer que \(\mathscr O_n(\mathbb R)\) (resp. \(\mathscr U_n(\mathbb C)\)) est un sous-groupe compact maximal dans \(GL_n(\mathbb R)\) (resp. \(GL_n(\mathbb C)\)) (commencer par montrer que les valeurs propres de tout élément d’un sous-groupe compact de \(GL_n(\mathbb K)\) sont de module \(1\)).



[ID: 2343] [Date de publication: 7 novembre 2022 22:38] [Catégorie(s): Algèbres bilinéaire et hermitienne ] [ Nombre commentaires: 0] [nombre d'éditeurs: 1 ] [Editeur(s): Emmanuel Vieillard-Baron ] [nombre d'auteurs: 1 ] [Auteur(s): Patrice Lassère ]
Accordéon
Titre
Solution
Texte

Quelques propriétés topologiques de \(\mathscr O_n(\mathbb R)\) et \(\mathscr U_n(\mathbb C)\)
Par Patrice Lassère le 7 novembre 2022 22:38
  1. \(\mathscr O_n(\mathbb R)\) est fermé dans \(M_n(\mathbb R)\) car \(\mathscr O_n(\mathbb R)=\varphi^{-1}(\{I_n\})\)\(\varphi\) est l’application continue sur \(M_n(\mathbb R)\) définie par \(A\mapsto \!^tAA\). \(\vert\vert\vert.\vert\vert\vert\) étant la norme sur \(M_n(\mathbb R)\) subordonnée à la norme euclidienne \(\Vert.\Vert\) de \(\mathbb R^n\), \(A\in\mathscr O_n(\mathbb R)\) implique \(\Vert AX\Vert=1\) et par suite \(\vert\vert\vert A\vert\vert\vert=1\). \(\mathscr O_n(\mathbb R)\) fermé bornée dans \(M_n(\mathbb R)\) est bien compact. La procédure est analogue pour \(\mathscr U_n(\mathbb C)\).

  2. On rappelle (voir ref. ????) que

  3. suivre......


Dans \(M_n(\mathbb R)\) : \(AB+A+B=0\quad\implies AB=BA\) *

7 novembre 2022 22:38 — Par Patrice Lassère

Soient \(A,B\in M_n(\mathbb R)\) telles que \(AB+A+B=0\) ; montrer que \(AB=BA\).



[ID: 2345] [Date de publication: 7 novembre 2022 22:38] [Catégorie(s): Algèbres bilinéaire et hermitienne ] [ Nombre commentaires: 0] [nombre d'éditeurs: 1 ] [Editeur(s): Emmanuel Vieillard-Baron ] [nombre d'auteurs: 1 ] [Auteur(s): Patrice Lassère ]
Accordéon
Titre
Solution
Texte

Dans \(M_n(\mathbb R)\) : \(AB+A+B=0\quad\implies AB=BA\)
Par Patrice Lassère le 7 novembre 2022 22:38

\(AB+A+B=0\) implique que \((A+I_n)(B+I_n)=AB+A+B+I_n=I_n\), les matrices \(A+I_n\) et \(B+I_n\) sont respectivement inverses l’une de l’autre. Par conséquent \((A+I_n)(B+I_n)=(B+I_n)(A+I_n)=I_n\), on développe et \(AB=BA\).


Dans \(M_n(\mathbb C)\), tout hyperplan rencontre \(GL_n(\mathbb C)\) (1) *

7 novembre 2022 22:38 — Par Patrice Lassère

[rms]-(2006).

Soit \(n\geq 2\).

  1. Montrer que si un hyperplan \(\mathscr H\) de \(M_n(\mathbb C)\) contient toutes les matrices nilpotentes, alors il contient une matrice inversible.

  2. Montrer que dans \(M_n(\mathbb C)\), tout hyperplan rencontre \(GL_n(\mathbb C)\).



[ID: 2347] [Date de publication: 7 novembre 2022 22:38] [Catégorie(s): Algèbres bilinéaire et hermitienne ] [ Nombre commentaires: 0] [nombre d'éditeurs: 1 ] [Editeur(s): Emmanuel Vieillard-Baron ] [nombre d'auteurs: 1 ] [Auteur(s): Patrice Lassère ]
Accordéon
Titre
Solution
Texte

Dans \(M_n(\mathbb C)\), tout hyperplan rencontre \(GL_n(\mathbb C)\) (1)
Par Patrice Lassère le 7 novembre 2022 22:38
  1. \(\mathscr H\) est un hyperplan de \(M_n(\mathbb C)\) : il existe donc une forme linéaire non nulle \(\varphi\) sur \(M_n(\mathbb C)\) telle que \(\mathscr H=\ker(\varphi)\). Tous les éléments \(E_{i,j}=((\delta_{i,j}^{k,l}))_{k,l}\) de la base canonique de \(M_n(\mathbb C)\) sont, si \(i\neq j\) des matrices nilpotentes donc, vu les hypothèses, dans \(\mathscr H\). Dans ce cas, la matrice \[A=E_{n,1}+\sum_{k=1}^{n-1} E_{k,k+1}= \begin{pmatrix} 0&1&0&\dots &0&\\ \vdots& \ddots&\ddots&\ddots&\vdots\\ \vdots& & \ddots&\ddots&0\\ 0& &&\ddots &1\\ 1&0&\dots&\dots&0 \end{pmatrix}\] appartient à \(\mathscr H\) comme combinaison linéaire d’éléments de \(\mathscr H\) mais visiblement \(A\in GL_n(\mathbb C)\) d’où le résultat.

  2. Soit \(\mathscr H=\ker(\varphi)\), \((\varphi\in M_n(\mathbb C)^\star\setminus\{0\})\) un hyperplan de \(M_n(\mathbb C)\).

    -Si \(\mathscr H\) contient toutes les matrices nilpotente, il n’y a rien à démontrer vu la question précédente.

    -Sinon, considérons une matrice nilpotente \(N\not\in\mathscr H\) i.e. \(\varphi(N)\neq 0\) et posons \[t:=\dfrac{\varphi(I_n)}{\varphi(N)}\in\mathbb C.\] Si \(\varphi(I_n)=0\) alors \(I_n\in\mathscr H\cap GL_n(\mathbb C)\) et le tour est joué. Sinon, puisque \(\varphi(I_n-tN)=0\) i.e. \(I_n-tN\in\mathscr H\), et comme \(N\) est nilpotente \((N^n=0)\) on peut écrire \[I_n=I_n-t^nN^n=(I_n-tN)\left(\sum_{k=0}^{n-1}t^kN^k\right)=\left(\sum_{k=0}^{n-1}t^kN^k\right)(I_n-tN).\] Autrement dit \(I_n-tN\) est inversible. CQFD.


Dans \(M_n(\mathbb C)\), tout hyperplan rencontre \(GL_n(\mathbb C)\) (2) *

7 novembre 2022 22:40 — Par Patrice Lassère

  1. Montrer que l’application qui à \(A\in M_n(\mathbb K)\ (\mathbb K=\mathbb R\ \text{ou}\ \mathbb C)\) associe \(f_A\ :\ M_n(\mathbb K)\ni M\mapsto f_A(M)=\text{Tr}(AM)\) établit un isomorphisme entre \(M_n(\mathbb K)\) et son dual.

  2. Soit \(f\in M_n(\mathbb K)'\) une forme linéaire sur \(M_n(\mathbb K)\) vérifiant \[f(XY)=f(YX),\qquad\forall\,X,Y\in M_n(\mathbb K).\] Montrer qu’il existe \(\lambda\in\mathbb K\) tel que pour toute matrice \(X\in M_n(\mathbb K)\), \(f(X)=\lambda\text{Tr}(X)\).

  3. Montrer que pour tout \(n\geq 2\), tout hyperplan de \(M_n(\mathbb K)\) rencontre \(GL_n(\mathbb K)\).



[ID: 2349] [Date de publication: 7 novembre 2022 22:40] [Catégorie(s): Algèbres bilinéaire et hermitienne ] [ Nombre commentaires: 0] [nombre d'éditeurs: 1 ] [Editeur(s): Emmanuel Vieillard-Baron ] [nombre d'auteurs: 1 ] [Auteur(s): Patrice Lassère ]
Accordéon
Titre
Solution
Texte

Dans \(M_n(\mathbb C)\), tout hyperplan rencontre \(GL_n(\mathbb C)\) (2)
Par Patrice Lassère le 7 novembre 2022 22:40
  1. On vérifie facilement que \(f_A\) est linéaire ; pour des raisons de dimension, il est donc suffisant de montrer que l’application \(A\mapsto f_A\) est injective :

    Soit \(A=((a_{i,j}))\) telle que \(f_A=0\). \((E_{i,j})\) désignant la base canonique de \(M_n(\mathbb K)\) on a pour \(1\leq k,l\leq n\) \[\begin{aligned} 0= f_A(E_{k,l})=\text{Tr}(AE_{k,l})&=\text{Tr}\left(\sum_{1\leq i,j\leq n}a_{i,j}E_{i,j}E_{k,l} \right)\\ &= \text{Tr}\left(\sum_{i=1}^na_{i,k}E_{i,l} \right)\quad\text{car}\ E_{i,j}E_{k,l}=\delta_{j,k}E_{i,l}\\ &=\sum_{i=1}^na_{i,k}\text{Tr}(E_{i,l})=a_{l,k}. \end{aligned}\] Et \(A\) est bien nulle.

  2. Soit \(f\) une telle forme et \(1\leq i,j\leq n\), si \(i\neq j\) \[f(E_{i,j})=f(E_{i,i}E_{i,j})=f(E_{i,j}E_{i,i})=0,\] et \[f(E_{i,i})=f(E_{i,j}E_{j,i})=f(E_{j,i}E_{i,j})=f(E_{j,j}):=\lambda.\] Ainsi, \(f\) et \(\lambda\text{Tr}\) coïncident sur la base canonique de \(M_n(\mathbb K)\) : elles sont égales.

    Remarque : On trouvera aussi dans [fgna1], exercice 7.7 une autre démonstration s’appuyant sur la première question.

  3. Un hyperplan \(H\) de \(M_n(\mathbb K)\) est le noyau d’une forme linéaire \(f\) non nulle ; il existe donc \(A\in M_n(\mathbb K)\) non nulle, telle que \(f=f_A\) et \(H=\{ X\in M_n(\mathbb K)\ :\ \text{Tr}(AX)=0\}\). Il s’agit donc de montrer qu’il existe \(X\in GL_n(\mathbb K)\) telle que \(\text{Tr}(AX)=0\). Pour cela, soit \(r\geq 1\) le rang de \(A\), il existe1 \(P\) et \(Q\) dans \(GL_n(\mathbb R)\) telle que \[PAQ=J_r=\begin{pmatrix} I_r & 0\\0&0 \end{pmatrix}.\] Alors, pour \(X\in M_n(\mathbb K)\), \(\text{Tr}(AX)=\text{Tr}(P^{-1}J_rQ^{-1}X)=\text{Tr}(J_rQ^{-1}XP^{-1})\) et il suffit donc de trouver une matrice inversible \(Y\) telle que \(\text{Tr}(J_rY)=0\) (\(X=QYP\) répondra alors à la question). Par exemple, la matrice de permutation \[Y=\begin{pmatrix} 0&0&\dots&\dots &0&1\\ 1&0&\ddots &&&0\\ 0&1&0&&&0\\ \vdots&&\ddots&\ddots&&\vdots\\ \vdots&&&\ddots&\ddots&\vdots\\ 0&\dots&\dots&\dots&1&0 \end{pmatrix}\] convient puisque \(J_rY\) est de diagonale nulle.

    Remarque :


  1. 1  donner une référence

Caractérisation des matrices nilpotentes par la trace *

7 novembre 2022 22:40 — Par Patrice Lassère

  1. Soit \(\mathbb K\) un sous-corps de \(\mathbb C\) et \(A\in M_n(\mathbb K)\). On suppose que pour tout \(k\in\mathbb N^\star\) la trace de \(A^k\) est nulle. Montrer que \(A\) est nilpotente.

  2. Soit \(A\in M_n(\mathbb C)\), établir l’équivalence des propriétés :

    1) La seule valeur propre de \(A\) est \(1\).

    2) \(\text{tr}(A)=\text{tr}(A^2)=\dots=\text{tr}(A^n)=n.\)



[ID: 2351] [Date de publication: 7 novembre 2022 22:40] [Catégorie(s): Algèbres bilinéaire et hermitienne ] [ Nombre commentaires: 0] [nombre d'éditeurs: 1 ] [Editeur(s): Emmanuel Vieillard-Baron ] [nombre d'auteurs: 1 ] [Auteur(s): Patrice Lassère ]
Accordéon
Titre
Solution
Texte

Caractérisation des matrices nilpotentes par la trace
Par Patrice Lassère le 7 novembre 2022 22:40

Nous restons ici fidèlement sur la trace de Francinou, Gianella et Nicolas [fgna2] qui proposent plusieurs solutions de ce classique problème.

  1.  -Sur \(\mathbb C\), le polynôme caractérique de \(A\) est scindé. Raisonnons par l’absurde en supposant \(A\) non nilpotente. Alors \(A\) possède au moins des valeurs propres non nulles que l’on note \(\lambda_1,\dots,\lambda_r,\ 1\leq r\leq n\) de multiplicités respectives \(n_1,\dots,n_r\). Par hypothèse nous avons pour tout \(k\in\mathbb N^\star\) : \[\rm{tr}(A^k)=n_1\lambda_1^k+\dots+n_r\lambda_r^k.\] Écrire ces relations pour \(k\) variant de \(1\) à \(r\) équivaut à dire que le vecteur \((n_1,\dots,n_r)\) est solution du système linéaire \[\begin{pmatrix} \lambda_1&\lambda_2&\dots&\lambda_r\\ \lambda_1^2&\lambda_2^2&\dots&\lambda_r^2\\ \vdots & & &\ldots\\ \lambda_1^r&\lambda_2^r&\dots&\lambda_r^r\\ \end{pmatrix}\begin{pmatrix} x_1\\x_2\\\ldots\\x_r \end{pmatrix}=0.\] Et ce système est de Cramer puisque le déterminant de la matrice du système vaut1 \[\lambda_1\dots\lambda_r\prod_{1\leq i<j\leq r}(\lambda_i-\lambda_j)\] donc nécessairement \(n_1=n_2=\dots=n_r=0\) ce qui est exclu.

    La seconde solution utilise les formules de Newton. Désignons cette fois-ci par \(\lambda_1,\dots,\lambda_n\) les racines du polynôme caractéristique \(\chi_A\) comptées avec leur multiplicités. Dire que \(\rm{tr}(A^k)=0\) pour tout entier \(k\in\mathbb N^\star\) revient exactement à dire que \[\rm{tr}(A^k)=0=\lambda_1^k+\dots+\lambda_n^k= 0,\quad\forall\,k\in\mathbb N^\star\] car \(A\) étant semblable à une matrice triangulaire à coefficients diagonaux \(\lambda_1,\dots,\lambda_n\) les matrices \(A^k\) sont elles semblables à des matrices triangulaires à coefficients diagonaux \(\lambda_1^k,\dots,\lambda_n^k\). Les formules de Newton2 impliquent alors que les fonctions symétriques élémentaires des racines \(\lambda_1,\dots,\lambda_n\) sont nulles : \[\sigma_1=\dots=\sigma_n=0.\] On en déduit que3 \[\chi_A=X^n-\sigma_1X^{n-1}+\dots+(-1)^{n-1}\sigma_{n-1}X+(-1)^n\sigma_n =X^n.\] Soit \(A^n=0\) d’aprés le théorème de Cayley-Hamilton

    Pour la troisième solution on procède par récurrence sur la taille de la matrice. Si \(n=1\), \(\rm{tr}(A)=0\) implique \(A=0\). Soit donc \(n\geq 2\), et supposons le résultat vrai jusqu’au rang \(n-1\). Le polynôme caractéristique de \(A\) \[\chi_A=X^n-\sigma_1X^{n-1}+\dots+(-1)^{n-1}\sigma_{n-1}X+(-1)^n\sigma_n\] vérifie par Cayley-Hamilton \[\chi_A(A)=0=A^n-\sigma_1A^{n-1}+\dots+(-1)^{n-1}\sigma_{n-1}A +(-1)^n\sigma_nI_n\] et en prenant la trace de cette dernière expression il vient \[(-1)^n\sigma_n\cdot n=(-1)^\lambda_1\dots\lambda_n n=(-1)^n n\det(A)=0\] soit \(\det(A)=0\). \(0\) est donc valeur propre de \(A\) et on peut écrire \[\chi_A=X^pQ\quad\text{avec}\quad Q(0)\neq 0\ \text{ et }\ p\geq 1.\] Le théorème de décomposition des noyaux assure alors que \[\mathbb K^n=\ker(\chi_A(A))=\ker(A^p)\oplus\ker(Q(A)).\] Supposons maintenant \(\ker(Q(A))\neq\{0\}\). Dans une base obtenue comme réunion d’une base de \(\ker(A^p)\) et d’une base de \(\ker(Q(A))\), l’endomorphisme \(A\) admet une matrice de la forme \[\begin{pmatrix} A'& 0\\0&B'\end{pmatrix}.\] Mais \(A'^p=0\), \(A'\) est donc nilpotente et par conséquent \(\rm{tr}(A'^k)=0,\ \forall\,k\in\mathbb N^\star\). Maintenant, comme \(A^k\) est semblable à \[\begin{pmatrix} A'^k& 0\\0&B'^k\end{pmatrix}\] on a nécessairement \(\rm{tr}(B'^k)=0,\ \forall\,k\in\mathbb N^\star\) : \(B'\) est aussi nilpotente. Mais par hypothèse, \(Q(0)\neq 0\) qui implique que la restriction de \(A\) à \(\ker(Q(A))\) est injective, soit \(B'\) inversible ce qui fournit la contradiction.

    On peut donc affirmer que \(\ker(Q(A))=\{0\}\), soit \(\mathbb K^n=\ker(A^p)\) et la matrice \(A\) est bien nilpotente.

  2. Soient \(\lambda_1,\dots,\lambda_n\) les valeurs propres de \(A\), comme \(\text{rang}(A^k)=\lambda_1^k+\dots+\lambda_n^k\) pour tout entier \(k\in\{1,\dots,n\}\), l’implication \((1)\Rightarrow (2)\) est immédiate.

    Pour \((2)\Rightarrow (1)\), remarquons que \(\text{spec}(A-I_n)=\{\,\lambda-1,\lambda\in \text{spec}(A)\}\), il est donc suffisant de montrer que la seule valeur propre de \(A-I_n\) est \(0\) ou encore que \(A-I_n\) est nilpotente.

    D’aprés la première question, \(A-I_n\) est nilpotente, si et seulement si, \(\text{tr}(A-I_n)^k=0\) pour tout entier \(k\in\{1,\dots,n\}\), et cette dernière vérification est élémentaire car : \[\text{tr}(A-I_n)^k=\sum_{l=0}^k\,\binom{k}{l}(-1)^l\text{tr}(A^{k-l}) =n\sum_{l=0}^k\,\binom{k}{l}(-1)^l=(1-1)^k=0.\]


  1. [fgna2], exercice 1.8.1  C’est le trés fameux déterminant de Vandermonde :
  2. [fgna1], exercice 5.26 où...2  Voir
  3. 3  où \((-1)^n\chi_A\) comme le veut souvent la coutume.

Sur l’équation \(A^p=I_n\) dans \(M_n(\mathbb Z)\). *

7 novembre 2022 22:40 — Par Patrice Lassère

Soient \(p,n,m\in\mathbb N^\star\) avec \(m\geq 2\). On considère une matrice \(A\in M_n(\mathbb Z)\) vérifiant \(A^p=I_n\) et \(A\equiv I_n(m)\). Montrer que \(A=I_n\).



[ID: 2353] [Date de publication: 7 novembre 2022 22:40] [Catégorie(s): Algèbres bilinéaire et hermitienne ] [ Nombre commentaires: 0] [nombre d'éditeurs: 1 ] [Editeur(s): Emmanuel Vieillard-Baron ] [nombre d'auteurs: 1 ] [Auteur(s): Patrice Lassère ]
Accordéon
Titre
Solution
Texte

Sur l’équation \(A^p=I_n\) dans \(M_n(\mathbb Z)\).
Par Patrice Lassère le 7 novembre 2022 22:40

Il existe par hypothèse \(B\in M_n(\mathbb Z)\) telle que \[A=I_n+mB.\] En outre, le polynôme \(X^p-1\) scindé à racines simples annule la matrice \(A\) qui est donc diagonalisable dans \(M_n(\mathbb C)\) ses valeurs propres étant des racines \(p\)-ièmes de l’unité.

Nous avons \(B=m^{-1}(A-I_n)\). \(B\) est aussi diagonalisable et ses valeurs propres sont de la forme \(m^{-1}(\lambda-1)\) avec \(\lambda^p=1\). Alors, comme \[\vert m^{-1}(\lambda-1)\vert \leq \dfrac{2}{m}<1\] on a \[\lim_{k\to+\infty} B^k=0.\] Mais les matrices \(B^k\) sont à coefficients entiers : la seule alternative est donc qu’il existe un entier \(k_0\) tel que \(B^{k_0}=0\). La matrice \(B\) est donc diagonalisable et nilpotente : c’est la matrice nulle et finalement \(A=I_n\)


Calcul de \(\exp(A)\)\(A=((\exp(2i\pi(k+l)/5)))_{k,l}\in M_5(\mathbb C)\). *

7 novembre 2022 22:40 — Par Patrice Lassère

Soit \(\omega=e^{2i\pi/5}\) et \(A=((\omega^{k+l})))_{0\leq k,l\leq 4}\in M_5(\mathbb C)\).

  1. \(A\) est-elle diagonalisable ?

  2. Calculer \(\exp(A)\).



[ID: 2355] [Date de publication: 7 novembre 2022 22:40] [Catégorie(s): Algèbres bilinéaire et hermitienne ] [ Nombre commentaires: 0] [nombre d'éditeurs: 1 ] [Editeur(s): Emmanuel Vieillard-Baron ] [nombre d'auteurs: 1 ] [Auteur(s): Patrice Lassère ]
Accordéon
Titre
Solution
Texte

Calcul de \(\exp(A)\)\(A=((\exp(2i\pi(k+l)/5)))_{k,l}\in M_5(\mathbb C)\).
Par Patrice Lassère le 7 novembre 2022 22:40
  1. Nous avons \[A=\begin{pmatrix}1&\omega&\dots&\omega^4\\ \omega&\omega^2&\dots&\omega^5\\ \omega^2&\omega^3&\dots&\omega^6\\ \vdots&\vdots&&\vdots\\ \omega^4&\omega^5&\dots&\omega^8 \end{pmatrix},\] les colonnes de \(A\) sont proportionnelles : \(A\) est donc de rang \(1\) et donc semblable à une matrice de la forme \[B=\begin{pmatrix}0&\dots&0&?\\ \vdots&&\vdots&\vdots\\ \vdots&&\dots&?\\ 0&\dots&0&\text{tr}(A)\end{pmatrix}.\] Mais \(\text{tr}(A)=1+\omega^2+\omega^4+\omega^6+\omega^8=0\) implique que \(B^2=0\) puis \(A^2=0\) : la matrice \(A\) n’est donc pas diagonalisable.

  2. \(A^2=0\) implique \(A^n=0\) pour tout entier \(n\geq 2\), par conséquent \(\exp(A)=I_5+A\).


Matrices entières inversibles *

7 novembre 2022 22:40 — Par Patrice Lassère

Soient \(A,B\in M_2(\mathbb Z)\) telles que \(A, A+B, A+2B, A+3B\) et \(A+4B\) soient inversibles à inverses dans \(M_2(\mathbb Z)\). Montrer que \(A+5B\) est inversible et que son inverse est encore à coefficients entiers.



[ID: 2357] [Date de publication: 7 novembre 2022 22:40] [Catégorie(s): Algèbres bilinéaire et hermitienne ] [ Nombre commentaires: 0] [nombre d'éditeurs: 1 ] [Editeur(s): Emmanuel Vieillard-Baron ] [nombre d'auteurs: 1 ] [Auteur(s): Patrice Lassère ]
Accordéon
Titre
Solution
Texte

Matrices entières inversibles
Par Patrice Lassère le 7 novembre 2022 22:40

Il faut se souvenir qu’une matrice \(M\) à coefficients entiers est inversible avec un inverse à coefficients entiers si et seulement si \(\det(M)=\pm 1\) (si \(M\) admet un tel inverse \(N\) alors, \(\det(M)\det(N)=\det(M)\det(N)=1\) soit \(\det(M)=\pm 1\) ; réciproquement si \(\det(M)=\pm 1\) alors \(\pm\widetilde{M}\) (où \(\widetilde{M}\) est la transposée des cofacteurs de \(M\)) est l’inverse de \(M\)).

Considérons alors le polynôme de degré au plus \(2\), \(f(x)=\det(A+xB)\). Vu les hypothèses et la remarque préliminaire \(f(x)=\pm 1\) pour \(x=0,1,2,3\) et \(4\) ; par le principe des tiroirs \(f\) prends une des valeurs \(\pm 1\) au moins trois fois ce qui le force à être constant, en particulier \(\det(A+5B)=\pm 1\), CQFD


Sur l’équation \(S=X^2\) dans \(M_n(\mathbb C)\) avec \(S\) symétrique et \(X\) antisymétrique. * Polytechnique

7 novembre 2022 22:40 — Par Patrice Lassère

Soit \(S\in M_n(\mathbb C)\) une matrice symétrique réelle, donner une condition nécessaire et suffisante sur ses valeurs propres pour quelle soit le carré d’une matrice antisymétrique réelle.



[ID: 2359] [Date de publication: 7 novembre 2022 22:40] [Catégorie(s): Algèbres bilinéaire et hermitienne ] [ Nombre commentaires: 0] [nombre d'éditeurs: 1 ] [Editeur(s): Emmanuel Vieillard-Baron ] [nombre d'auteurs: 1 ] [Auteur(s): Patrice Lassère ]
Accordéon
Titre
Solution
Texte

Sur l’équation \(S=X^2\) dans \(M_n(\mathbb C)\) avec \(S\) symétrique et \(X\) antisymétrique.
Par Patrice Lassère le 7 novembre 2022 22:40

On va montrer que \(S\) est le carré d’une matrice antisymétrique réelle si et seulement si, toutes ses valeurs propres sont négatives et ses valeurs propres strictement négatives sont de multiplicité paire.

-(condition suffisante). Si \(S\) est la matrice nulle, alors \(S=0^2\) et on peut supposer désormais \(S\neq 0\). \(S\) est symétrique réelle, ses valeurs propres sont réelles et peuvent, vu les hypothèses, s’écrire sous la forme \[-a_1^2,-a_1^2,-a_2^2,-a_2^2,\dots,-a_l^2,-a_l^2,0,\dots,0\]\(a_1,a_2,\dots a_l\in\mathbb R_+^\star\) (et ne sont pas forcément distincts). Toujours parce que \(S\) est symétrique réelle, elle est diagonalisable dans une base orthonormée : \[\exists\,O\in O_n(\mathbb R)\ :\quad O^{-1}SO= \text{diag}(-a_1^2,-a_1^2,-a_2^2,-a_2^2,\dots,-a_l^2,-a_l^2,0,\dots,0).\] Notons pour \(k\in\{1,\dots,l\}\) \[A_k=\begin{pmatrix}0&-a_k\\a_k&0\end{pmatrix}\] et considérons la matrice diagonale par blocs \[R=\text{diag}(A_1,A_2,\dots,A_l,0,\dots,0)\in M_n(\mathbb R).\] \(R\) est bien antisymétrique réelle et un calcul par blocs montre que \[R^2= \text{diag}(-a_1^2,-a_1^2,-a_2^2,-a_2^2,\dots,-a_l^2,-a_l^2,0,\dots,0)=O^{-1}SO=\,^t\!OSO,\] \(OR\, ^t\!O\) est une matrice antisymétrique réelle vérifiant \((OR\, ^t\!O)^2=S\), CQFD.

-Pour la condition nécessaire, si \(S=R^2\) avec \(R\) antisymétrique, comme \(R=-\,^t\!R\) on a \(S=R^2=-\,^t\!RR\) qui implique \[\forall\, X\in\mathbb R^n,\quad ^t\!XSX=-\,^tX\,^t\!RRX=-\Vert RX\Vert^2\] qui prouve que \(S\) est symétrique réelle négative : son spectre est donc inclu dans \(\mathbb R_-\).

Les valeurs propres de \(S\) sont les carrés des valeurs propres de \(R\) qui sont donc imaginaire pures et stables par conjugaison puisque \(R\) est à coefficients réels. (\(\lambda\in\text{spec}(S)\implies \pm i\sqrt{\lambda}\in\text{spec}(R)\)) : elles sont donc de multiplicité paire. CQFD.


Convexité, matrice symétrique, calcul d’intégrale *

7 novembre 2022 22:40 — Par Patrice Lassère

Montrer que l’application \(A\mapsto (\det(A))^{-1/2}\) de l’ensemble \(\mathscr S_n^{++}\) des matrices symétriques définies positives dans \(\mathbb R_+^\star\) est strictement convexe.



[ID: 2361] [Date de publication: 7 novembre 2022 22:40] [Catégorie(s): Algèbres bilinéaire et hermitienne ] [ Nombre commentaires: 0] [nombre d'éditeurs: 1 ] [Editeur(s): Emmanuel Vieillard-Baron ] [nombre d'auteurs: 1 ] [Auteur(s): Patrice Lassère ]
Accordéon
Titre
Solution
Texte

Convexité, matrice symétrique, calcul d’intégrale
Par Patrice Lassère le 7 novembre 2022 22:40

Soit \(A\in\mathscr S_n^{++}\) de valeurs propres \(\lambda_1,\dots,\lambda_n\). En se placant dans une base orthonormale de réduction de \(A\) nous avons \[\int_{\mathbb R^n}e^{-\langle Ax,x\rangle}dx_1\dots dx_n =\int_{\mathbb R^n}e^{-(\lambda_1y_1^2+\dots+\lambda_ny_n^2)}dy_1\dots dy_n =(\det(A))^{-1/2}C\quad\text{où}\quad C=\pi^{n/2}.\] Il suffit maintenant de remarquer que la fonction \(x\mapsto e^{-x}\) est strictement convexe sur \(\mathbb R\) pour conclure.


Matrices symétriques *

7 novembre 2022 22:40 — Par Patrice Lassère

Pour \(A=((a_{i,j}))\in M_n(\mathbb R),\ (n\in\mathbb N^\star)\) symétrique et vérifiant \(A^2=A\), établir les inégalités suivantes :

  1. \(\displaystyle 0\leq\sum_{1\leq i,j\leq n}a_{i,j}\leq n.\)

  2. \(\displaystyle \sum_{1\leq i,j\leq n}\vert a_{i,j}\vert\leq n\sqrt{\text{rang}(A)}\).

  3. \(\displaystyle \sum_{1\leq i,j\leq n}\vert a_{i,j}\vert< n^{3/2}\) si \(n\geq 2\).



[ID: 2363] [Date de publication: 7 novembre 2022 22:40] [Catégorie(s): Algèbres bilinéaire et hermitienne ] [ Nombre commentaires: 0] [nombre d'éditeurs: 1 ] [Editeur(s): Emmanuel Vieillard-Baron ] [nombre d'auteurs: 1 ] [Auteur(s): Patrice Lassère ]
Accordéon
Titre
Solution
Texte

Matrices symétriques
Par Patrice Lassère le 7 novembre 2022 22:40
  1. Notons \(V= \,^t \!(1,1,\dots,1)\in M_{n,1}(\mathbb R)\), par un calcul élémentaire

    \[\sum_{1\leq i,j\leq n}a_{i,j}=\,^t\!VAV=\,^tVA^2V=\,^tV(\,^t\!A A)V=\Vert AV\Vert^2\geq 0.\]

    Pour l’inégalité de droite, remarquons que \(B=((b_{i,j}))= I_n-A\) vérifie encore \(^t\!B=B\) et \(B^2=B\), si bien que

    \[\sum_{1\leq i,j\leq n}b_{i,j}= n-\sum_{1\leq i,j\leq n}a_{i,j} \geq 0.\]

  2. Notons \(A'=((\vert a_{i,j}\vert))\in M_n(\mathbb R)\) et \(U=(V,V,\dots, V)\in M_n(\mathbb R)\). \(M_n(\mathbb R)\) étant muni de sa structure euclidienne canonique \(\langle A,B\rangle =\text{trace}(\,^t\!A B)\), par Cauchy-Schwarz

    \[\langle A', U\rangle^2 \leq \Vert A'\Vert^2\,\Vert U\Vert^2\]

    soit

    \[\left(\sum_{1\leq i,j\leq n}\vert a_{i,j}\vert\right)^2 \leq \left(\sum_{1\leq i,j\leq n} a_{i,j}^2\right)\left(\sum_{1\leq i,j\leq n} 1^2 \right) (\bigstar)\]

    mais

    \[\sum_{1\leq i,j\leq n} a_{i,j}^2=\text{trace}(\,^t\!A A)=\text{trace}(A)\]

    et puisque \(A\) est une matrice de projection \(\text{trace}(A)=\text{rang}(A)\leq n\) soit avec \((\bigstar)\)

    \[\left(\sum_{1\leq i,j\leq n}\vert a_{i,j}\vert\right)^2\leq n^2\text{rang}(A)\leq n^3.\]

  3. Vu l’inégalité précédente, on aura toujours une inégalité large, et l’égalité équivaut à \(\text{rang}(A)=n\) soit \(A\in GL_n(\mathbb R)\) puis (\(A^2=A\)) \(A=I_n\), mais alors

    \[\sum_{1\leq i,j\leq n}\vert a_{i,j}\vert = n<n^{3/2}\quad\text{dès que}\quad n\geq 2,\]

    d’où le résultat.


Espaces euclidiens et projection orthogonale *

7 novembre 2022 22:40 — Par Patrice Lassère

\(\mathscr S_n\) désignant le sous-espace dans \(M_n(\mathbb R)\) des matrices symétriques réelles, calculer pour \(A=((a_{ij}))\in M_n(\mathbb R)\)

\[\inf_{M=((m_{ij}))\in \mathscr S_n}\,\sum_{1\leq i,j\leq n}\left(a_{ij}-m_{ij}\right)^2\]



[ID: 2365] [Date de publication: 7 novembre 2022 22:40] [Catégorie(s): Algèbres bilinéaire et hermitienne ] [ Nombre commentaires: 0] [nombre d'éditeurs: 1 ] [Editeur(s): Emmanuel Vieillard-Baron ] [nombre d'auteurs: 1 ] [Auteur(s): Patrice Lassère ]
Accordéon
Titre
Solution
Texte

Espaces euclidiens et projection orthogonale
Par Patrice Lassère le 7 novembre 2022 22:40

Il est bien entendu équivalent de déterminer

\[\inf_{M=((m_{ij}))\in \mathscr S_n}\,\sqrt{\sum_{1\leq i,j\leq n}\left(a_{ij}-m_{ij}\right)^2},\qquad{(\text{$\star$})}\]

et si \(M_n(\Bbb R)\) est muni du produit scalaire

\[\langle A,B\rangle=\text{tr}(A\,^t\!B)\]

on peut reécrire (\(\star\)) sous la forme

\[\inf_{M\in \mathscr S_n}\,\sqrt{\sum_{1\leq i,j\leq n}\left(a_{ij}-m_{ij}\right)^2}=\inf_{M\in \mathscr S_n}\,\text{tr}\left((A-M)\,^t\!(A-M)\right)=\inf_{M\in \mathscr S_n}\Vert A-M\Vert=\text{dist}(A,\mathscr S_n).\]

Par le théorème de projection orthogonale \[\text{dist}(A,\mathscr S_n)=\Vert A-B\Vert\]\(B\) est la projection orthogonale de \(A\) sur \(\mathscr S_n\), le théorème de projection nous assure en outre que \(B-A\in\mathscr S_n^\perp\) et il est bien connu que \(\mathscr S_n^\perp\) n’est rien d’autre que l’ensemble des matrices antisymétriques, soit

\[^t\!(A-B)= B-A\quad\iff\quad B= ^t\!B={1\over 2}(A+ ^t\!A)\]

si bien que

\[\inf_{M=((m_{ij}))\in \mathscr S_n}\,\sum_{1\leq i,j\leq n}\left(a_{ij}-m_{ij}\right)^2 =\sum_{1\leq i,j\leq n}\dfrac{\left(a_{ij}-a_{ji}\right)^2}{2}.\]


Une matrice symétrique non diagonalisable *

7 novembre 2022 22:41 — Par Patrice Lassère

La matrice symétrique \(\displaystyle\begin{pmatrix} 2&i\\ i&0\end{pmatrix}\) est-elle diagonalisable ?



[ID: 2367] [Date de publication: 7 novembre 2022 22:41] [Catégorie(s): Algèbres bilinéaire et hermitienne ] [ Nombre commentaires: 0] [nombre d'éditeurs: 1 ] [Editeur(s): Emmanuel Vieillard-Baron ] [nombre d'auteurs: 1 ] [Auteur(s): Patrice Lassère ]
Accordéon
Titre
Solution
Texte

Une matrice symétrique non diagonalisable
Par Patrice Lassère le 7 novembre 2022 22:41

Non bien sûr ! \(\lambda=1\) est son unique valeur propre : si elle était diagonalisable elle serait semblable et donc égale à \(I_2\).

Remarques : -un contre-exemple à toujours avoir sous la main : ce sont les matrices symétriques réelles qui sont toujours diagonalisables et dans une base orthonormée s’il vous plaît.

-Profitons en pour donner l’exemple d’une matrice non triangularisable (sur \(\mathbb R\) bien entendu...) : \(\begin{pmatrix} 1&-1\\1&1\end{pmatrix}\) et d’une matrice non diagonalisable (sur \(\mathbb C\) bien entendu...), à savoir \(\begin{pmatrix} 1&1\\0&1\end{pmatrix}\).

-Dans la même veine, pour terminer voici deux matrices \[\begin{pmatrix} 1&1&0&0\\0&1&0&0\\0&0&1&1\\0&0&0&1\end{pmatrix},\qquad \begin{pmatrix} 1&0&0&0\\0&1&0&0\\0&0&1&1\\0&0&0&1\end{pmatrix}\] qui ne sont pas semblables mais qui ont même polynômes minimal et caractéristiques. Vous pouvez remarquer que ce sont des matrices \(4\times 4\), ce qui est normal car pour \(n\leq 3\), deux matrices (ou endomorphismes) sont semblables si, et seulement si, ils ont mêmes polynômes minimal et caractéristiques. Mais pour \(n\geq 4\) l’implication non triviale est fausse.


Produit scalaire, continuité, topologie *

7 novembre 2022 22:41 — Par Patrice Lassère

Soit \(E\) un espace pré-hilbertien, montrer que l’ensemble \[\mathscr O:=\left\{ (x,y)\in E\times E\text{ tels que } x,y \text{ sont libres dans }E\right\}\] est un ouvert de \(E\times E\).



[ID: 2369] [Date de publication: 7 novembre 2022 22:41] [Catégorie(s): Algèbres bilinéaire et hermitienne ] [ Nombre commentaires: 0] [nombre d'éditeurs: 1 ] [Editeur(s): Emmanuel Vieillard-Baron ] [nombre d'auteurs: 1 ] [Auteur(s): Patrice Lassère ]
Accordéon
Titre
Solution
Texte

Produit scalaire, continuité, topologie
Par Patrice Lassère le 7 novembre 2022 22:41

L’application \[\varphi\ :\ (x,y)\in E\times E\mapsto \varphi(x,y)=\langle x,y\rangle-\sqrt{\langle x,x\rangle\langle y,y\rangle}\in\mathbb R.\] est continue sur \(E\times E\) muni de sa topologie naturelle d’espace produit (la continuité du produit scalaire résulte de l’inégalité de Cauchy-Schwarz) et, vu le cas d’égalité dans l’inégalité de Cauchy-Schwarz \(\mathscr O=\varphi^{-1}(\mathbb R^\star)\). C’est donc un ouvert comme image réciproque d’un ouvert par une application continue.


Autour de la trace *

7 novembre 2022 22:41 — Par Patrice Lassère

Soit \(A\in M_n(\mathbb R)\) telle que \[\forall \,X\in M_n(\mathbb R),\ \text{tr}(X)=0\Longrightarrow \text{tr}(AX)=0.\] Montrer qu’il existe \(\lambda\in\mathbb R\) tel que \(A=\lambda I_n\).



[ID: 2371] [Date de publication: 7 novembre 2022 22:41] [Catégorie(s): Algèbres bilinéaire et hermitienne ] [ Nombre commentaires: 0] [nombre d'éditeurs: 1 ] [Editeur(s): Emmanuel Vieillard-Baron ] [nombre d'auteurs: 1 ] [Auteur(s): Patrice Lassère ]
Accordéon
Titre
Solution
Texte

Autour de la trace
Par Patrice Lassère le 7 novembre 2022 22:41

Muni du produit scalaire \(\langle A,B\rangle=\text{tr}(A\,{ }^t\!B)\), \(M_n(\mathbb R)\) est un espace euclidien et

\[E:=\{\,A\in M_n(\mathbb R)\ :\ \text{tr}(A)=0\}\]

est un sous-espace vectoriel de \(M_n(\mathbb R)\) de dimension \(n^2-1\) : c’est en effet le noyau de la forme linéaire

\[\varphi\ :\ A\in M_n(\mathbb R)\mapsto \varphi(A)=\text{tr}(A)\]

donc \(\dim E+\dim \left(\text{im}(\varphi)\right) =n^2\Longrightarrow \dim E=n^2-\dim \left(\text{im}(\varphi)\right)=n^2-1\) car \(\text{im}(\varphi)\) est un sous-espace non réduit à l’origine de \(\mathbb R\) donc égal à \(\mathbb R\).

Par suite \(\dim E^\perp =1\) et \(E^\perp =\mathbb RI_n\) car bien sûr \(I_n\in E^\perp\), mais

\[\left(\forall \,X\in M_n(\mathbb R),\ \text{tr}(X)=0\Longrightarrow \text{tr}(AX)=0=\langle A,{ }^t\!X\rangle\right)\iff\left(A\in E^\perp=\mathbb RI_n\right)\]


Bases orthormées *

7 novembre 2022 22:41 — Par Patrice Lassère

Soient \((e_1,\dots,e_d)\), \((f_1,\dots,f_d)\) deux bases orthonormées d’un espace euclidien \((E,\langle.\vert.\rangle)\). Si \(u\in\mathscr L(E)\), montrer que la quantité \[\sum_{1\leq i,j\leq d}\langle u(e_i),f_j\rangle^2{\text{($\star$)}}\] est indépendante du choix de ces deux bases.



[ID: 2373] [Date de publication: 7 novembre 2022 22:41] [Catégorie(s): Algèbres bilinéaire et hermitienne ] [ Nombre commentaires: 0] [nombre d'éditeurs: 1 ] [Editeur(s): Emmanuel Vieillard-Baron ] [nombre d'auteurs: 1 ] [Auteur(s): Patrice Lassère ]
Accordéon
Titre
Solution
Texte

Bases orthormées
Par Patrice Lassère le 7 novembre 2022 22:41

\((f_j)_1^d\) étant orthonormée, on a avec Pythagore \[\sum_{1\leq i,j\leq d}\langle u(e_i),f_j\rangle^2=\sum_{i=1}^d \sum_{j=1}^d \langle u(e_i),f_j\rangle^2 =\sum_{i=1}^d \Vert u(e_i)\Vert^2,\] nous sommes donc déja assurés que (\(\star\)) ne dépend pas \((f_j)_1^d\). Il reste à prouver que \[\sum_{i=1}^d \Vert u(e_i)\Vert^2=\sum_{i=1}^d \Vert u(f_i)\Vert^2\] Pour cela si \(A=((a_{i,j}))\) désigne la matrice de \(u\) dans la base \((e_i)_1^d\) \[\rm{trace}(A\,\!^tA)=\sum_{1\leq i,j\leq d}a_{i,j}^2=\sum_{i=1}^d \Vert u(e_i)\Vert^2\] de même \[\sum_{i=1}^d \Vert u(f_i)\Vert^2=\rm{trace}(B\,\!^tB)\]\(B=\rm{mat}(u,(f_j)_1^d)=P^{-1}AP\) et \(P\in GL_d(\mathbb R)\) est la matrice de passage entre les deux bases. Ces deux bases étant orthonormées, \(P\) est orthogonale i.e. \(P^{-1}=\,\!^tP\) et par suite \[\rm{trace}(B\,\!^tB)=\rm{trace}(P^{-1}AP\,\!^t(P^{-1}AP)=\rm{trace}(P^{-1}AP\,\!^tP\,\!^tA\,\!^tP) =\rm{trace}(P^{-1}A\,\!^tAP)=\rm{trace}(A\,\!^tA).\]


Toute matrice carrée réelle est produit de deux matrices symétriques réelles *

7 novembre 2022 22:41 — Par Patrice Lassère

  1. Montrer que pour toute matrice carrée réelle, il existe une matrice de passage à sa transposée qui soit symétrique.

  2. En déduire que toute matrice carrée réelle est le produit de deux matrices symétriques réelles.



[ID: 2375] [Date de publication: 7 novembre 2022 22:41] [Catégorie(s): Algèbres bilinéaire et hermitienne ] [ Nombre commentaires: 0] [nombre d'éditeurs: 1 ] [Editeur(s): Emmanuel Vieillard-Baron ] [nombre d'auteurs: 1 ] [Auteur(s): Patrice Lassère ]
Accordéon
Titre
Solution
Texte

Toute matrice carrée réelle est produit de deux matrices symétriques réelles
Par Patrice Lassère le 7 novembre 2022 22:41
  1. La solution repose sur le fait suivant : toute matrice \(A\in M_n(\mathbb R)\) est semblable à sa transposée mais si de plus \(A\) est une matrice cyclique (i.e. semblable a une matrice compagnon), on peut imposer à la matrice de passage d’être symétrique réelle. que l’on va pouvoir étendre à tout \(M_n(\mathbb R)\).

    En effet, soient \[A\,=\,\begin{pmatrix} 0 & 0 & \dots & 0 & -a_0 \\ 1 & 0 & \dots & 0 & -a_1 \\ 0 & 1 & \dots & 0 & -a_2 \\ \vdots & \ddots & \ddots & \vdots & \vdots \\ 0 & \dots & 0 & 1 & -a_{n-1} \end{pmatrix},\ \ S\,=\, \begin{pmatrix} a_{1} & a_{2} & \dots & a_{n-1} & 1 \\ a_{2} & a_{3} & \dots & 1 & 0 \\ \vdots & \vdots & \dots & \vdots & \vdots \\ a_{n-1} & 1&\dots &0&0\\ 1 & 0 &\dots & 0 & 0 \end{pmatrix}\in GL_n(\Bbb C),\] alors \[AS\,=\,\begin{pmatrix} -a_0 & 0 & 0 & \dots & 0 &0\\ 0 & a_2 & a_3 & \dots & a_{n-1} &1\\ 0 & a_3 & a_4 & \ddots &1 &0 \\ \vdots & \vdots & \vdots & \dots & \vdots &\vdots\\ 0 & a_{n-1} &1&\dots &0&0\\ 0 & 1 & 0 & \dots & 0&0 \end{pmatrix}\] est une matrice symétrique et par conséquent \(AS={ }^t\!\big(AS\big)=S\big({ }^t\!A\big)\) i.e. \(A=S\big({ }^t\!\!A\big)S^{-1}\).

    Il en résulte immédiatement que toute matrice de Frobenius (i.e. une matrice constituée de blocs diagonaux cycliques) \(F\) admet une matrice de passage à sa transposée \(S\) symétrique réelle (\(F=SAS^{-1}\)). Vérifions maintenant que cette propriété se généralise à toutes les matrices.

    Comme toute matrice \(A\in M_n(\mathbb R)\) est semblable à une matrice de Frobenius (c’est le théorème de décomposition de Frobenius analogue cyclique du théorème de décomposition de Jordan) il existe donc \(P\in GL_n(\mathbb R), S\in \mathscr S_n(\mathbb R)\cap GL_n(\mathbb R)\) et \(F\) matrice de Frobenius, telles que \[A= P F P^{-1},\quad F=S\,{ }^t\!F S^{-1}.\] Ainsi \[\begin{aligned}A= P (S\,{ }^t\!F S^{-1}) P^{-1}&= (P S)( \,{ }^t\!P \,{ }^t\!P^{-1}) \,{ }^t\!F (\,{ }^t\!P \,{ }^t\!P^{-1 })(S^{-1} P^{-1})\\ &=(P S \,{ }^t\!P) (\,{ }^t\!P^{-1} \,{ }^t\!F \,{ }^t\!P)( \,{ }^t\!P^{-1 }S^{-1} P^{-1})\\ &=(P S \,{ }^t\!P) \,{ }^t\!A( \,{ }^t\!P^{-1 }S^{-1} P^{-1})\\ &=(P S \,{ }^t\!P) \,{ }^t\!A (P S \,{ }^t\!P)^{-1}=S_1 \,{ }^t\!A S_1^{-1} \end{aligned}\]\(S_1= P S \,{ }^t\!P\). La matrice de passage \(S_1\) est clairement symétrique, nous avons donc démontré que pour toute matrice \(A\in M_n(\mathbb R)\), il existe une matrice de passage symétrique \(S\) telle que \(A=S \,{ }^t\!A S^{-1}\).

  2. On peut alors écrire \[A=S S',\quad \text{où}\quad S'= \,{ }^t\!A S^{-1}\] et comme \[\,{ }^t\!(S')=\,{ }^t\!(\,{ }^t\!A S^{-1})=\,{ }^t\! S^{-1}A= S^{-1}S \,{ }^t\!A S^{-1}=\,{ }^t\!A S^{-1}=S'\] \(S'\) est symétrique et le tour est joué puisque \(A=S S'\).

    Remarques : -On peut consulter1 The factorisation of a square matrix into two symmetric matrices Amer.Math.Monthly 1986-6, page 462/64 pour une approche via la decomposition de Jordan et apprendre aussi que ce resultat est du a Frobenius lui même.

    -Dans [grula], on trouve cette jolie caractérisation des matrices diagonales réelles : Une matrice \(A\in M_n(\mathbb R)\) est diagonalisable si, et seulement si, il existe une matrice \(S\) symétrique définie positive telle que \({ }^t\!A=S^{-1}A{ }^t\!S\).

    La preuve n’est pas difficle : si \(A\) est diagonalisable, il existe \(P\in GL_n(\mathbb R)\) telle que \(P^{-1}AP=D\)\(D\) est diagonale. Donc \(D={ }^t\!D={ }^t\!P{ }^t\!A{ }^t\!P^{-1}\), soit \({ }^t\!P{ }^t\!A{ }^t\!P^{-1}=P^{-1}AP\) où encore \({ }^t\!A=(P{ }^t\!P)^{-1}A(P{ }^t\!P)\) et la condition est nécessaire puisque \(S=P{ }^t\!P\) est symétrique définie positive.

    Réciproquement, supposons qu’il existe \(S\) symétrique définie positive (vérification immédiate) telle que \({ }^t\!A=S^{-1}AS\) ; \(S\) définie positive se factorise2 sous la forme \(S=P{ }^t\!P\)\(P\in GL_n(\mathbb R)\), donc \({ }^t\!A={ }^t\!P^{-1}P^{-1}AP{ }^t\!P\) soit \({ }^t\!P{ }^t\!A{ }^t\!P^{-1}=P^{-1}AP\) et finalement \({ }^t\!(P^{-1}AP)=P^{-1}AP\). Ainsi, \(P^{-1}AP\) est symétrique réelle, donc diagonalisable qui entraine à son tour \(A\) diagonalisable.


  1. 1  Une référence aimablement communiquée par notre collégue J.B.Hiriart-Urruty (Université Toulouse 3).
  2. Il existe \(O\in\mathscr O_n(\mathbb R)\) telle que \(S=O^{-1}DO={ }^t\!ODO\) avec \(D=\text{diag}(\lambda_1,\dots,\lambda_n),\ (\lambda_i>0,\ \forall\,i)\) ; si on pose \(\sqrt{D}=\text{diag}(\sqrt{\lambda_1},\dots,\sqrt{\lambda_n})\) alors \(P=\sqrt{D}O\) convient.↩︎


Accordéon
Titre
Solution
Texte

Sur l’équation \(\det(I_n-xA-yB)=\det(I_n-xA)\det(I_n-yB),\quad \forall\,x,y\in\mathbb R\).
Par Patrice Lassère le 7 novembre 2022 22:41

On procède par récurrence sur la taille \(n\) de la matrice, la propriété est clairement vraie pour \(n=1\).

Soit \(n\geq 2\), on suppose la propriété vraie au rang \(n-1\) et soient \(A,B\in\mathscr S_n(\mathbb R)\) telles que \[\det(I_n-xA-yB)=\det(I_n-xA)\det(I_n-yB),\quad \forall\,x,y\in\mathbb R.{(H_n)}\] Montrons que \(AB=0\). Tout repose sur le fait suivant (on exclut le cas trivial où au moins une des deux matrices est nulle) :

\((\bigstar)\quad\) \(B\) (ou \(A\), c’est pareil par symétrie) admet une valeur propre \(\lambda\) non nulle telle que \(\ker(A)\cap\ker (I_n-\lambda B)\neq \{0\}\).

Admettons pour le moment \((\bigstar)\) et prouvons l’assertion au rang \(n\). Soit \(\lambda\) une valeur propre non nulle de \(B\) telle que \(\ker(A)\cap\ker (I-\lambda B)\neq \{0\}\) et soit \(e_\lambda\in\ker(A)\cap\ker (I-\lambda B)\setminus\{0\}\). On considère alors une base orthogonale \(\mathscr B\) de \(\mathbb R^n\) de premier terme \(e_\lambda\). Les matrices de \(A\) et \(B\) dans cette base sont respectivement de la forme \(\left( {\begin{array}{c|c} 0 & 0 \\ \hline 0& A' \end{array}} \right)\) et \(\left( {\begin{array}{c|c} \lambda & 0 \\ \hline 0& B' \end{array}} \right)\) où les matrices \(A'\) et \(B'\) sont dans \(\mathscr S_{n-1}(\mathbb R)\) (c’est la symétrie de \(A\) et \(B\) et l’orthogonalité de \(\mathscr B\) qui imposent aux deux matrices d’etre symétrique ce qui impose à son tour la symétrie des deux sous matrices \(A'\) et \(B'\) et les zéros sur les premières lignes à partir du second terme). Maintenant par un calcul élémentaire \[\begin{aligned} \det(I_n-xA-yB)&=(1-\lambda y)\det(I_{n-1}-xA'-xB')\\ \det(I_n-xA)\det(I_n-yB)&=(1-\lambda y)\det(I_{n-1}-xA')\det(I_{n-1}-xB') \end{aligned}\] soit \[\det(I_{n-1}-xA'-xB')=\det(I_{n-1}-xA')\det(I_{n-1}-xB'),\quad \forall\,x,y\in\mathbb R.\] On peut donc appliquer l’hypothèse de récurrence au rang \(n-1\) : \(A'B'=0\). De là \[AB=P^{-1} \left( {\begin{array}{c|c} 0 & 0 \\ \hline 0& A' \end{array}} \right)\left( {\begin{array}{c|c} \lambda & 0 \\ \hline 0& B' \end{array}} \right) P=P^{-1}\left( {\begin{array}{c|c} 0 & 0 \newline \hline 0& A'B' \end{array}} \right)P=0\] et le tour est joué.\(\blacksquare\)

Il reste donc à établir la propriété \((\bigstar)\), c’est la partie délicate qui se déduit des deux lemmes suivants :

Soit \(A=((a_{ij}))\) une matrice symétrique positive ; si un élément diagonal \(a_{ii} \ (1\leq i\leq n)\) est nul il en est de même pour la ligne (et la colonne correspondante) i.e. \(a_{ij}=a_{ji}=0,\ \forall\,1\leq j\leq n\).

Preuve du lemme 1 : Considérons une telle matrice et supposons par l’absurde qu’il existe un coefficient \(a_{ji}\neq 0\). Soit \(X_t=(x_k)_1^n\) le vecteur colonne où \(x_j=1, x_i=ta_{ji},\ t\in\mathbb R\) et où les autres composantes sont nulles, alors \(^t\! X_tUX_t=a_{jj}+ta_{ji}^2\) change de signe lorsque \(t\) décrit \(\mathbb R\) ce qui est absurde.\(\blacksquare\)

Soient \(U,V\in M_n(\mathbb R)\). On suppose \(U\geq 0,\ V\) symétrique et \[\forall\,t\in\mathbb R\ :\quad \det(U-tV)=0.\] Alors \[\ker(V)\cap\ker(U)\neq\{0\}.\]

Preuve du lemme 2 : On diagonalise \(V\) dans une base orthonormée : \[PV ^t\! P=\text{Diag}(\lambda_1,\dots,\lambda_r,0,\dots,0)=\left( {\begin{array}{c|c} D_r & 0 \\ \hline 0&0 \end{array}} \right):=D,\quad \lambda_i\neq 0,\] on a alors \[PU ^t\! P-tPV ^t\! P=\left({\begin{array}{c|c} U_1 & U_2 \\ \hline ^t\! U_2&U_3 \end{array}}\right)-t\left( {\begin{array}{c|c} D_r & 0 \\ \hline 0&0 \end{array}} \right)=\left({\begin{array}{c|c} U_1-tD_r & U_2 \\ \hline ^t\! U_2&U_3 \end{array}}\right).\] Vu l’hypothèse, le polynôme en "\(t\)" \[\det(PU ^t\! P-tD)=0,\quad\forall\,t\in\mathbb R\] est identiquement nul (bien entendu, \(r<n\)) et son terme de plus haut degré est (au signe prés) \(\det(U_3)\) qui est donc nul.

Quitte à modifier les \(n-r\) vecteurs de base \(e_{r+1},\dots,e_n\) on peut supposer \(U_3\) diagonale, plus précisément soit \(Q\in\mathscr O_{n-r}\) telle que \[^t\! QU_3Q=\text{diag}(u_{r+1,r+1},\dots,u_{n,n})\] Dans la nouvelle base correspondant à la matrice orthogonale \(\left({\begin{array}{c|c} I_r& 0 \\ \hline 0& Q \end{array}}\right)\) notre matrice \(PU ^t\! P-tPV ^t\! P\) s’écrit \(\left({\begin{array}{c|c} U_1-tD_r & U_2' \\ \hline ^t\! U_2'& \text{diag}(u_{r+1,r+1},\dots,u_{n,n}) \end{array}}\right)\)

Il en résute que \(\det(U_3)=(-1)^ru_{r+1,r+1},\dots,u_{n,n}=0\) qui assure que la matrice \(U\) symétrique positive admet un élément diagonal disons \(u_{ii}, (i\in\{r+1,\dots,n\})\) nul ; on peut donc appliquer le lemme 1 et affirmer que la ième colonne correspondante dans \(U\) est nulle. Ainsi \(PU ^t\! Pe_i=0,\) mais on a aussi \[PV ^t\! Pe_i=De_i=\left( {\begin{array}{c|c} D_r & 0 \\ \hline 0&0 \end{array}} \right)e_i=0\] car \(i\in\{r+1,\dots,n\}\) ou encore \(U ^t\! Pe_i=V ^t\! Pe_i=0,\) soit finalement \[^t\! Pe_i\in\ker(U)\cap\ker(V).\] CQFD\(\blacksquare\)

Preuve de \((\bigstar)\) : Soit \(\lambda\) une valeur propre non nulle de \(B\), l’hypothèse \((H_n)\) implique que \[\det(I_n-\lambda^{-1}B-xA)=0,\quad\forall\,x\in\mathbb R\] on est donc dans le cadre du lemme 2 avec \(V=A\) et \(U=I_n-\lambda^{-1}B\) qui sera positive si on choisit pour \(\lambda\) la plus grande des valeurs propres non nulle.

C’était un exercice estival aimablement proposé par notre collégue J.B.Hiriart-Urruty.


;
Success message!